GI Pathology Amboss Q&A

Ace your homework & exams now with Quizwiz!

A previously healthy 75-year-old man comes to the physician with a 6-month history of fatigue, weight loss, and abdominal pain. He drinks 2 oz of alcohol on the weekends and does not smoke. He is retired but previously worked in a factory that produces plastic pipes. Abdominal examination shows right upper quadrant tenderness; the liver edge is palpable 2 cm below the ribs. A liver biopsy specimen shows pleomorphic spindle cells that express PECAM-1 on their surface. Which of the following is the most likely diagnosis? This patient's symptoms together with his history of working at a plastic factory suggests a type of liver cancer associated with occupational exposure to vinyl chloride. Vinyl chloride is used to make various polyvinyl chloride (PVC) plastic products, including pipes.

Angiosarcoma Hepatic angiosarcomas are the third most common type of primary hepatic malignancy, and their occurrence is associated with exposure to vinyl chloride, which is likely an occupational hazard in this patient, as well as arsenic and thorium dioxide. Hepatic angiosarcomas have a poor prognosis due to their high recurrence rate and resistance to chemotherapy and radiotherapy. Immunophenotyping of tumor cells in hepatic angiosarcoma will be positive for PECAM-1 (also known as CD31), a vascular antigen that indicates the presence of endothelial cells.

A 48-year-old woman comes to the emergency department because of a 1-day history of fever, vomiting, and abdominal pain. Two weeks ago, while visiting Guatemala, she had an emergency appendectomy under general inhalational anesthesia. During the surgery, she received a transfusion of 1 unit of packed red blood cells. Her temperature is 38.3°C (100.9°F) and blood pressure is 138/76 mm Hg. Examination shows jaundice and tender hepatomegaly. Serum studies show: Alkaline phosphatase 132 U/L Aspartate aminotransferase 760 U/L Bilirubin Total 3.8 mg/dL Direct 3.1 mg/dL Anti-HAV IgG positive Anti-HAV IgM negative Anti-HBs positive HBsAg negative Anti-HCV antibodies negative Abdominal ultrasonography shows an enlarged liver. A biopsy of the liver shows massive centrilobular necrosis. Which of the following is the most likely underlying cause of this patient's condition? After undergoing surgery, this patient developed jaundice, fever, vomiting, and tender hepatomegaly, with highly elevated serum transaminases, which indicates acute postoperative hepatitis.

Adverse effect of anesthetic This patient's acute postoperative hepatitis is highly suggestive of halothane hepatitis. Halothane hepatitis commonly leads to elevated serum aminotransferases, elevated bilirubin, eosinophilia, and slightly elevated alkaline phosphatase, as seen in this patient. This patient's symptoms began 13 days following surgery, which is consistent with the timeframe for postoperative onset of halothane hepatitis (2 days to 3 weeks). Liver biopsy is not necessary for diagnosis but typically shows massive centrilobular hepatic necrosis.

A 59-year-old man comes to the emergency department because of progressive abdominal swelling and shortness of breath for 1 week. He drinks 12 to 13 alcoholic beverages daily. He appears emaciated. Examination shows pallor, jaundice, hepatomegaly, gynecomastia, and a protuberant abdomen with a fluid wave and shifting dullness. Periodic monitoring of which of the following markers is most appropriate for this patient? This patient most likely has alcoholic liver cirrhosis (pallor, jaundice, hepatomegaly, gynecomastia, ascites, history of alcohol use disorder) and is at an increased risk of developing hepatocellular carcinoma.

Alpha-fetoprotein Periodic monitoring of alpha-fetoprotein along with abdominal ultrasound is the most appropriate surveillance method for hepatocellular carcinoma; approximately 80% of cases of HCC occur in patients with liver cirrhosis. The combined use of AFP and abdominal ultrasound increases detection rates of HCC compared to the use of ultrasound alone. AFP can also be used as a marker for yolk sac tumors (endodermal sinus tumors), mixed germ cell tumors, and ataxia telangiectasia.

A 38-year-old man is admitted to the hospital because of fever, yellowing of the skin, and nausea for 1 day. He recently returned from a backpacking trip to Brazil and Paraguay, during which he had a 3-day episode of high fever that resolved spontaneously. Physical examination shows jaundice, epigastric tenderness, and petechiae over his trunk. Five hours after admission, he develops dark brown emesis and anuria. Despite appropriate lifesaving measures, he dies. Postmortem liver biopsy shows eosinophilic degeneration of hepatocytes with condensed nuclear chromatin. This patient's hepatocytes were most likely undergoing which of the following processes? This patient passed the three clinical stages of yellow fever (infection, remission, intoxication), which is endemic in tropical regions of South America. Liver biopsy shows Councilman bodies, which are typical for yellow fever.

Apoptosis Apoptosis is the process through which hepatocytes respond to certain hepatotoxic insults such as acute viral hepatitis or yellow fever (as seen here). On liver biopsy, apoptotic hepatocytes appear shrunken and deeply eosinophilic with condensed chromatin (pyknosis) and a fragmented nucleus (karyorrhexis).

A 49-year-old woman comes to the physician with a 2-month history of mild abdominal pain, nausea, and several episodes of vomiting. She often feels full after eating only a small amount of food. Abdominal examination shows mild right upper quadrant tenderness and a liver span of 16 cm. Ultrasonography shows a 5 x 4 cm hyperechoic mass in the left lobe of the liver. The mass is surgically excised. A photomicrograph of the resected specimen is shown. Which of the following is the most likely diagnosis? The photomicrograph shows numerous cystic spaces lined by a single layer of endothelium. The cysts contain erythrocytes and multiple thromboses. There is no evidence of normal hepatic parenchyma.

Cavernous hemangioma This patient's biopsy specimen shows a benign vascular tumor, which confirms the diagnosis of cavernous hepatic hemangioma. Cavernous hemangioma of the liver is the most common benign hepatic tumor and is typically seen in women aged 30-50 years. While most patients with hepatic hemangiomas are asymptomatic, a large hepatic hemangioma can compress the adjacent bowel (causing early satiety) and stretch the Glisson capsule (causing upper abdominal pain). Because hepatic hemangiomas are highly vascular, they typically appear as hyperechoic masses on ultrasonography. Needle aspiration or biopsy should be avoided in the diagnostic workup of hepatic hemangiomas due to the risk of hemorrhage.

A 45-year-old man comes to the physician because of a 5-day history of fever, malaise, and right upper abdominal pain. Examination of the abdomen shows tenderness in the right upper quadrant. His leukocyte count is 18,000/mm3 (92% neutrophils) and serum alkaline phosphatase is 130 U/L. Ultrasonography of the abdomen shows a 3-cm hypoechoic lesion in the right lobe of the liver with a hyperemic rim. Which of the following is the most likely underlying cause of this patient's condition? A triad of fever, malaise, and right upper abdominal pain in a patient with neutrophilic leukocytosis and a hepatic lesion with a hyperemic rim (due to surrounding edema and inflammation) on ultrasonography is diagnostic of a liver abscess.

Cholangitis Ascending infection from a biliary tract pathology, such as cholangitis, is the most common cause of hepatic abscesses. The infection is typically caused by bacteria and mainly is polymicrobial, with E. coli, Klebsiella pneumoniae, and Streptoccocus spp. being among the most frequent causal pathogens. Hepatic abscesses may also result from intraabdominal infections (e.g., acute appendicitis, peritonitis), or hematogenous spread from systemic circulation (e.g., sepsis).

A previously healthy 67-year-old man comes to the physician because of a history of recurrent right lower abdominal pain for the past 2 years. A CT scan shows a 1.2-cm (0.47-in) mass located in the terminal ileum. He undergoes surgical removal of the mass. A photomicrograph of the resected specimen is shown. Cells from this tissue are most likely to stain positive for which of the following? The photomicrograph shows a tissue specimen of a neuroendocrine origin consistent with a carcinoid tumor.

Chromogranin A Chromogranin A is a protein that is found in neuroendocrine cells, which are mainly present in the pancreas (i.e., islet cells), gastrointestinal tract, lungs, and adrenal glands. The biopsy of this patient's ileal mass shows numerous small cells of similar shape and size (monomorphic cells) arranged in a rosette pattern that is typical of a carcinoid tumor. Chromogranin A, synaptophysin, and neuron-specific enolase can be used as an immunostain to confirm the neuroendocrine origin of this tumor

A 49-year-old woman is admitted to the hospital for the evaluation of postprandial colicky pain in the right upper quadrant of the abdomen. Abdominal ultrasound shows multiple round, hyperechoic structures within the gallbladder lumen. She undergoes a cholecystectomy. A photograph of the content of her gallbladder is shown. This patient is most likely to have which of the following additional conditions? The image shows black pigment gallstones.

Chronic hemolytic anemia Black pigment gallstones are associated with chronic hemolytic anemia (e.g., due to hereditary spherocytosis or beta thalassemia major), in which an elevated red blood cell breakdown leads to an increased serum concentration of indirect bilirubin. Excess bilirubin is excreted by the liver into the bile. The excess bilirubin then precipitates within the gallbladder due to supersaturation and reacts with calcium salts to form black calcium bilirubinate stones. Black pigment stones account for approx. 10% of all gallstones.

A 63-year-old man comes to the physician with a 4-week history of fatigue, crampy abdominal pain, watery diarrhea, and pain in his mouth and gums. He returned from a 2-week trip to the Dominican Republic 2 months ago. He has smoked one pack of cigarettes daily for 45 years. Examination shows three 1.5-cm, painful ulcers in the mouth. Abdominal examination shows mild tenderness to palpation in the right lower quadrant without guarding or rebound. His hemoglobin concentration is 11.2 g/dL, mean corpuscular volume is 75 fL, and leukocyte count is 11,900 mm3. Colonoscopy shows a cobblestone mucosa. A photomicrograph of a biopsy specimen is shown. Which of the following is the most likely diagnosis? Colonoscopy findings in patients with this condition can also show transmural inflammation, linear ulcers, fistulas, and aphthous hemorrhagic mucosa.

Crohn disease Crohn disease (CD) has a bimodal distribution and can develop in older patients around the age of 60 years. CD typically presents with chronic, watery, nonbloody diarrhea and abdominal pain, mainly in the right lower quadrant (RLQ). Extraintestinal manifestations, such as the oral aphthae in this patient, are often also seen. Laboratory findings include microcytic anemia and leukocytosis. CD is strongly associated with a history of smoking. Biopsy results showing granulomatous inflammation of the colonic mucosa are diagnostic of Crohn disease.

A 34-year-old woman with Crohn disease comes to the physician because of a 4-week history of nausea, bloating, and epigastric pain that occurs after meals and radiates to the right shoulder. Four months ago, she underwent ileocecal resection for an acute intestinal obstruction. An ultrasound of the abdomen shows multiple echogenic foci with acoustic shadows in the gallbladder. Which of the following mechanisms most likely contributed to this patient's current presentation? In this patient who had ileocecal resection for complicated Crohn disease, the clinical features (nausea, bloating, and epigastric pain) and ultrasonographic findings (gallbladder echogenic foci with an acoustic shadow) indicate a diagnosis of cholelithiasis. Enterohepatic circulation is impaired in patients with Crohn disease and ileocecal resection.

Decreased biliary concentration of bile acids A decreased concentration of bile acids in the bile can occur in patients with Crohn disease. Bile acids are normally reabsorbed in the terminal ileum and returned to the liver for reuse via the enterohepatic circulation. However, bile acid malabsorption occurs in patients with Crohn disease due to ileitis and/or terminal ileum resection as seen in this case. A decreased bile acid to cholesterol ratio causes cholesterol supersaturation and predisposes to cholesterol gallstone formation, which can lead to cholelithiasis and cholecystitis.

A 67-year-old woman comes to the physician because of a 5-day history of episodic abdominal pain, nausea, and vomiting. She has coronary artery disease and type 2 diabetes mellitus. She takes aspirin, metoprolol, and metformin. She is 163 cm (5 ft 4 in) tall and weighs 91 kg (200 lb); her BMI is 34 kg/m2. Her temperature is 38.1°C (100.6°F). Physical examination shows dry mucous membranes, abdominal distension, and hyperactive bowel sounds. Ultrasonography of the abdomen shows air in the biliary tract. This patient's symptoms are most likely caused by obstruction at which of the following locations? This patient has risk factors for gallstone disease (e.g., obesity, age > 40, female gender). In a patient with signs of intestinal obstruction (e.g., vomiting, abdominal pain, abdominal distension, hyperactive bowel sounds), the presence of pneumobilia is highly suggestive of gallstone ileus.

Distal ileum Gallstone ileus is a rare complication of cholecystitis in which a cholecystoenteric fistula forms, through which air can enter the biliary tree. Gallstones can also pass through into the bowel lumen, leading to bowel obstruction and gallstone ileus. The distal ileum is separated from the large intestine by the ileocecal valve, which is the narrowest part of the small intestine and hence the most likely location for gallstone obstruction. Other causes of distal ileal obstruction include fecaliths, Meckel diverticulum, and intussusception.

A 46-year-old man comes to the physician for a follow-up examination. Two weeks ago, he underwent laparoscopic herniorrhaphy for an indirect inguinal hernia. During the procedure, a black liver was noted. He has a history of intermittent scleral icterus that resolved without treatment. Serum studies show: Aspartate aminotransferase 30 IU/L Alanine aminotransferase 35 IU/L Alkaline phosphatase 47 mg/dL Total bilirubin 1.7 mg/dL Direct bilirubin 1.1 mg/dL Which of the following is the most likely diagnosis? This patient has predominantly direct hyperbilirubinemia, which indicates defective excretion of conjugated bilirubin into the bile.

Dubin-Johnson syndrome Dubin-Johnson syndrome is caused by a hereditary defect in a transport protein of the biliary canaliculi. This defect impairs conjugated bilirubin excretion and causes direct hyperbilirubinemia. This defect also impairs the excretion of metabolites of epinephrine, which subsequently give the liver a black appearance. Dubin-Johnson syndrome is a benign condition that does not require treatment, as mild to moderate jaundice is typically the only clinical sign.

A female newborn delivered at 38 weeks' gestation is evaluated for abdominal distention and bilious vomiting 24 hours after delivery. The pregnancy and delivery were uncomplicated. She appears lethargic and her fontanelles are sunken. An x-ray of the abdomen is shown. This infant most likely has a congenital obstruction affecting which of the following anatomic structures? This condition is strongly associated with trisomy 21.

Duodenum This patient has congenital duodenal atresia, which causes bowel obstruction just distal to the major duodenal ampulla leading to abdominal distension, bilious vomiting, and features of dehydration (e.g., sunken anterior fontanelle). Her x-ray shows the classic "double bubble sign" of air in a distended stomach and proximal duodenum, which are separated by the pylorus, with no gas in the more distal bowel. Approximately 25% of cases are associated with trisomy 21.

A 56-year-old man comes to the physician because of intermittent retrosternal chest pain. Physical examination shows no abnormalities. Endoscopy shows salmon pink mucosa extending 5 cm proximal to the gastroesophageal junction. Biopsy specimens from the distal esophagus show nonciliated columnar epithelium with numerous goblet cells. Which of the following is the most likely cause of this patient's condition? Esophageal mucosa is normally whitish-pink and composed of stratified squamous epithelium. Intestinal mucosa is normally composed of columnar epithelium with goblet cells.

Esophageal exposure to gastric acid Chronic exposure of the esophagus to gastric acid promotes transformation of the stratified squamous epithelium of the esophagus to metaplastic columnar epithelium with goblet cells (Barrett esophagus). Barrett esophagus classically occurs in patients with GERD, which commonly presents with intermittent retrosternal pain, as seen in this patient. Adequate diagnosis of Barrett esophagus requires histologic confirmation of intestinal metaplasia.

A 57-year-old man comes to the physician because of progressively worsening epigastric pain and 6.8-kg (15-lb) weight loss for the past 2 months. Two weeks ago, he noticed painful, red blotches on the medial aspect of his right calf. He has smoked one pack of cigarettes daily for the last 35 years. He appears thin. Physical examination shows tender, erythematous nodules on the right ankle and left antecubital fossa. Endoscopy shows a large mass in the antrum of the stomach. A biopsy specimen of the gastric mass shows disorganized, mucin-secreting cells with surrounding fibrosis. These cells most likely originated from which of the following structures? This patient's cancer is associated with an increased CA 19-9 and is typically diagnosed at a late stage because it does not cause symptoms until it has metastasized.

Exocrine ducts in the body of the pancreas The most likely diagnosis in this patient is pancreatic adenocarcinoma, the most common type of pancreatic cancer. The glandular mucin-secreting cells are typical of pancreatic adenocarcinoma, which is derived from exocrine cells of the pancreas. The pancreatic head is the most common site for adenocarcinoma, but cancers that originate there often manifest with pruritus and painless jaundice secondary to obstruction of the common bile duct. Since this patient lacks these symptoms, his cancer more likely originates in the body or tail of the pancreas. No matter where the cancer starts, pancreatic adenocarcinoma does not typically manifest until after it has metastasized, as seen in this patient.

A 38-year-old man comes to the physician because of an 8-month history of upper abdominal pain. During this period, he has also had nausea, heartburn, and multiple episodes of diarrhea with no blood or mucus. He has smoked one pack of cigarettes daily for the past 18 years. He does not use alcohol or illicit drugs. Current medications include an antacid. The abdomen is soft and there is tenderness to palpation in the epigastric and umbilical areas. Upper endoscopy shows several ulcers in the duodenum and the upper jejunum as well as thick gastric folds. Gastric pH is < 2. Biopsies from the ulcers show no organisms. Which of the following tests is most likely to confirm the diagnosis? Several peptic ulcers, ulcers in unusual locations (i.e., jejunum), and thick gastric folds should raise suspicion for Zollinger-Ellison syndrome.

Fasting serum gastrin level Determining the fasting serum gastrin level is the best initial test for diagnosing gastrinoma (Zollinger-Ellison syndrome), which this patient likely has. A 10-fold increase in gastrin levels is conclusive evidence of a gastrinoma, especially in combination with a gastric pH < 2. A secretin stimulation test (SST) may also be used in the diagnosis of ZES. However, it must be performed without acid suppression therapy and is not recommended in patients with severe manifestations of ZES.

A 65-year-old man is brought to the emergency department because of a 1-day history of fever and disorientation. His wife reports that he had abdominal pain and diarrhea the previous day. He drinks 60 oz of alcohol weekly. His pulse is 110/min and blood pressure is 96/58 mm Hg. Examination shows jaundice, palmar erythema, spider nevi on his chest, dilated veins on the anterior abdominal wall, and 2+ edema of the lower extremities. The abdomen is soft and diffusely tender; there is shifting dullness to percussion. His albumin is 1.4 g/dL, bilirubin is 5 mg/dL, and prothrombin time is 31 seconds (INR = 3.3). Hepatitis serology is negative. A CT scan of the abdomen is shown. Which of the following processes is the most likely explanation for these findings? This patient presents with features of spontaneous bacterial peritonitis (e.g., fever, diffuse abdominal tenderness, diarrhea), likely due to advanced alcoholic cirrhosis. Cirrhosis is confirmed by the CT scan, which shows an atrophic nodular liver, an enlarged spleen, and ascites.

Fibrous bands surrounding regenerating hepatocytes Hepatocyte destruction and hepatic stellate cell activation via inflammatory cytokines lead to excess collagen fiber production in the periportal and pericentral zones. The continuous formation of fibrous bands surrounding regenerating hepatocytes (i.e., fibrosis and nodular regeneration) destroy the normal architecture of the liver and result in the loss of normal liver metabolic function. Connective tissue formation in the periportal zones and other fibrotic changes lead to an increased intrahepatic vascular tone and subsequently, to portal hypertension, manifesting with splenomegaly, caput medusae, and/or esophageal varices.

A previously healthy, 24-year-old man comes to the physician because of a 6-week history of loose, nonbloody stools. He also reports abdominal pain, intermittent nausea, and fever. He has not had vomiting, tenesmus, or rectal pain. His vital signs are within normal limits. Abdominal examination reveals tenderness of the right lower abdomen without rebound tenderness. Rectal exam is unremarkable. Laboratory studies show a leukocyte count of 14,800/mm3 and an erythrocyte sedimentation rate of 51 mm/h. Test of the stool for occult blood and stool studies for infection are negative. A CT scan of the abdomen shows transmural thickening and surrounding fat stranding of discrete regions of the terminal ileum and transverse colon. A colonoscopy is performed and biopsy specimens of the affected areas of the colon are taken. Which of the following findings is most specific for this patient's condition? This young man has abdominal pain, diarrhea, fever, laboratory evidence of inflammation, and a CT scan of the abdomen showing skip lesions in the colon with transmural inflammation and sparing of the rectum. Given that the infectious workup was negative, autoimmune enteritis is high on the differential. Crohn disease is more likely than ulcerative colitis based on the CT scan and the absence of tenesmus, gross bleeding, and rectal pain.

Formation of noncaseating granulomas Formation of noncaseating granulomas is a histopathological finding in ∼ 30% of patients with Crohn disease (CD). While the absence of granulomas does not rule out CD, their presence helps to distinguish CD from ulcerative colitis. This distinction can be challenging, especially in CD involving the colon. Distinguishing between the two conditions is crucial, however, as their management differs significantly. On biopsy, the presence of granulomatous inflammation, transmural inflammation, fissures, and aphthous ulcers would strongly suggest CD.

A 52-year-old woman comes to the emergency department because of epigastric abdominal pain that started after her last meal and has become progressively worse over the past 6 hours. She has had intermittent pain similar to this before, but it has never lasted this long. Her temperature is 39°C (102.2°F). Examination shows a soft abdomen with normal bowel sounds. The patient has sudden inspiratory arrest during right upper quadrant palpation. Her alkaline phosphatase, total bilirubin, amylase, and aspartate aminotransferase levels are within the reference ranges. Abdominal imaging is most likely to show which of the following findings? This middle-aged woman presents with post-prandial epigastric abdominal pain, fever, and a positive Murphy sign, suggesting a diagnosis of acute cholecystitis.

Gallstone in the cystic duct The vast majority of cases of acute cholecystitis are due to obstructing gallstones in the cystic duct. In a patient with clinical features of acute cholecystitis (e.g., RUQ/epigastric pain lasting over 6 hours, fever, and positive Murphy sign), ultrasonography of the RUQ is indicated. Findings that support the diagnosis include gallbladder wall thickening, gallbladder wall edema, obstructing gallstone, and/or a positive Murphy sign on ultrasound. Gallstones are a common incidental finding on ultrasound but in this setting would also suggest acute cholecystitis.

A 55-year-old man comes to the physician because of a 3-week history of intermittent burning epigastric pain. His pain improves with antacid use and eating but returns approximately 2 hours following meals. He has a history of chronic osteoarthritis and takes ibuprofen daily. Upper endoscopy shows a deep ulcer located on the posterior wall of the duodenal bulb. This ulcer is most likely to erode into which of the following structures? The structure most at risk of damage lies directly posterior to the first part of the duodenum (the duodenal bulb).

Gastroduodenal artery The gastroduodenal artery runs directly posterior to the first part of the duodenum after originating from the common hepatic artery. It supplies the pylorus, proximal duodenum, and head of the pancreas. A peptic ulcer located on the posterior wall of the duodenal bulb can erode into the gastroduodenal artery and cause significant gastrointestinal bleeding. Bleeding is the most common complication of peptic ulcer disease.

A 61-year-old man comes to the physician because of a 6-month history of epigastric pain and a 9-kg (20-lb) weight loss. He feels full and bloated even after eating small portions of food. His hemoglobin concentration is 9.5 g/dL with a mean corpuscular volume of 78 μm3. Test of the stool for occult blood is positive. Esophagogastroduodenoscopy shows a 2-cm raised lesion with central ulceration on the lesser curvature of the stomach. Histologic examination of a gastric biopsy specimen from the lesion is most likely to show which of the following? Epigastric pain, weight loss, early satiety, and microcytic anemia in a patient with a suspicious lesion in the lesser curvature of the stomach should raise suspicion for gastric cancer. Adenocarcinoma is by far the most common type of gastric cancer.

Gland-forming columnar cells Gland-forming columnar cells are seen in adenocarcinoma, which is the most common type of gastric cancer (accounting for approx. 90% of cases). The endoscopic finding of a suspicious ulcer confined to the lesser curvature is consistent with intestinal (type I) gastric adenocarcinoma, which is characterized by aggressive local spread and typically metastasizes to the liver and regional lymph nodes. Risk factors include chronic H. pylori infection, smoking, and chronic gastritis.

A 10-year-old girl is brought to the emergency department because of a 2-day history of bloody diarrhea and abdominal pain. Four days ago, she visited a petting zoo with her family. Her temperature is 39.4°C (102.9°F). Abdominal examination shows tenderness to palpation of the right lower quadrant. Stool cultures at 42°C grow colonies that turn black after adding phenylenediamine. Which of the following best describes the most likely causal organism? This girl presents with right lower quadrant pain, bloody diarrhea, and fever after exposure to animals. Many pathogens can cause this presentation, but stool cultures that grow oxidase-positive organisms (colonies that turn black with phenylenediamine) at 42°C suggests infection with Campylobacter species.

Gram-negative, flagellated bacteria that do not ferment lactose Campylobacter jejuni is a species of comma-shaped, non-lactose fermenting bacteria with polar flagella. It is a leading cause of bloody diarrhea in children. The infection is transmitted fecal-orally by ingestion of undercooked, contaminated poultry/meat, unpasteurized milk, or fomites containing the pathogen. Direct contact with infected animals (dogs, cats, pigs) at petting zoos is a risk factor. Campylobacter infection can lead to Guillain-Barré syndrome and reactive arthritis. Salmonella spp. are also gram-negative, flagellated bacteria that do not ferment lactose and infection with Salmonella can cause fever, abdominal pain, and bloody diarrhea in younger children. Exposure to reptiles and live poultry is also a potential risk factor for transmission. However, Salmonella spp. test negative for oxidase.

A 75-year-old man comes to the physician because of 3 weeks of vague abdominal pain and nausea. He has a history of recurrent peptic ulcer disease and received a blood transfusion 10 years ago for severe anemia. An upper gastrointestinal endoscopy is performed, and a biopsy of the stomach shows diffuse infiltrates of small lymphoid cells that are positive for CD20 antigen. He is diagnosed with extranodal marginal zone lymphoma of the mucosa-associated lymphoid tissue, and treatment is initiated. Which of the following is the most likely causal pathogen? The causal pathogen can be seen using the Giemsa stain.

Helicobacter pylori Helicobacter pylori is a gram-negative, comma-shaped bacterium that mainly infects the gastric mucosa. Infection with H. pylori is the primary risk factor for developing mucosa-associated lymphoid tissue (MALT) lymphoma, which this patient has. More than 90% of patients with gastric MALT lymphoma have H. pylori infection. This malignancy is effectively prevented and treated using triple therapy: a combination of amoxicillin, clarithromycin, and a PPI (e.g., esomeprazole). H. pylori is also a risk factor for gastric adenocarcinoma (mainly the intestinal type) and peptic ulcer disease. Eradication of the pathogen also decreases the risk of developing these diseases.

A 31-year-old man comes to the physician because of a 2-day history of nausea, abdominal discomfort, and yellow discoloration of the eyes. Six weeks ago, he had an episode of fever, joint pain, swollen lymph nodes, and an itchy rash on his trunk and extremities that persisted for 1 to 2 days. He returned from a backpacking trip to Colombia two months ago. His temperature is 39°C (101.8°F). Physical examination shows scleral icterus. Infection with which of the following agents is the most likely cause of this patient's findings? This patient's current condition (jaundice, nausea) suggest liver pathology and his medical history suggests a serum sickness-like reaction (fever, arthritis, itchy rash).

Hepatitis B Hepatitis B is associated with a serum sickness-like reaction, which typically occurs during the prodromal period of the infection 1-2 weeks after antigen exposure and is thought to be mediated by immune complex formation. Symptoms of serum sickness-like reaction usually resolve within a few weeks, but symptoms from acute HBV infection may persist for longer, as seen in this patient. Other causes of serum sickness-like reaction include antibiotics (cefaclor, penicillin), streptococcal infection, and vaccines.

A 65-year-old man comes to the physician for a routine health maintenance examination. He has a strong family history of colon cancer. A screening colonoscopy shows a 4 mm polyp in the upper sigmoid colon. Which of the following findings on biopsy is associated with the lowest potential for malignant transformation into colorectal carcinoma? Small epithelial proliferation in the rectosigmoid colon has the lowest potential for malignant transformation.

Hyperplastic epithelium at the base of crypts Hyperplastic polyps consist of hyperplastic epithelium at the base of the crypts of Lieberkuhn. They are a common type of polyp and most frequently occur in the rectosigmoid and left colon. Hyperplastic polyps are benign and carry a very low risk of transforming into malignancy because there is no dysplasia present. Other examples of polyps that are benign include mucosal polyps and submucosal polyps.

A 3-day-old girl is brought to the physician by her mother because of difficulty feeding and lethargy for 1 day. She had jaundice after birth and was scheduled for a follow-up visit the next day. Her hemoglobin is 18.5 g/dL, total bilirubin is 38.1 mg/dL, and direct bilirubin is 0.1 mg/dL. Despite appropriate measures, the infant dies. At autopsy, examination of the brain shows deep yellow staining of the basal ganglia and subthalamic nuclei bilaterally. Which of the following is the most likely cause of this infant's findings? This patient's symptoms and autopsy findings of deep yellow staining in the basal ganglia are highly suggestive of kernicterus due to unconjugated hyperbilirubinemia.

Impaired glucuronidation of bilirubin Impaired glucuronidation of bilirubin leads to elevated unconjugated bilirubin. Concurrent with pathological neonatal jaundice, normal conjugated bilirubin levels and the absence of anemia, the patient's condition is highly suggestive of type 1 Crigler-Najjar syndrome, in which unconjugated bilirubin crosses the blood-brain barrier, leading to kernicterus and, potentially, death.

A 2-week-old infant is brought to the emergency room because of 4 episodes of bilious vomiting and inconsolable crying for the past 3 hours. Abdominal examination shows no abnormalities. An upper GI contrast series shows the duodenojejunal junction to the right of the vertebral midline; an air-filled cecum is noted in the right upper quadrant. Which of the following is the most likely cause of this patient's condition? The duodenojejunal junction normally lies to the left of the midline, adjacent to the vertebral bodies.

Incomplete intestinal rotation Incomplete intestinal rotation, indicated by a malpositioned duodenojejunal junction (usually located to the left of the midline) and cecum (normally located in the right lower quadrant), can manifest as duodenal obstruction, causing bilious vomiting without abdominal distension, as seen in this infant. Obstruction is due to an extrinsic duodenal compression by peritoneal bands (Ladd bands) that cross over the duodenum to fix the abnormal cecum to the liver or peritoneum.

A 47-year-old man with alcoholic cirrhosis comes to the physician for a follow-up examination. Examination of the skin shows erythema over the thenar and hypothenar eminences of both hands. He also has numerous blanching lesions over the trunk and upper extremities that have a central red vessel with thin extensions radiating outwards. Which of the following is the most likely underlying cause of these findings? This patient's skin findings are consistent with palmar erythema and spider angiomas, which are related to his cirrhosis.

Increased circulating estrogen Hyperestrogenism in patients with liver dysfunction occurs due to impaired hepatic metabolism of estrogen and androstenedione (which is then converted to estrogen by aromatase in adipose cells). Elevated estrogen levels stimulate the proliferation and dilation of capillaries, resulting in palmar erythema and spider angioma. In men, increased estrogen levels can also cause feminization, which may manifest as gynecomastia, decreased body hair (e.g., loss of chest, pubic, and/or axillary hair), and/or testicular atrophy.

A 2720-g (6-lb) female newborn delivered at 35 weeks' gestation starts vomiting and becomes inconsolable 48 hours after birth. The newborn has not passed her first stool yet. Examination shows abdominal distention and high-pitched bowel sounds. A water-soluble contrast enema study shows microcolon. Serum studies show increased levels of immunoreactive trypsinogen. Which of the following is the most likely additional laboratory finding? This newborn's inability to pass her first stool (most likely caused by meconium ileus) and increased serum immunoreactive trypsinogen are indicative of cystic fibrosis as the underlying disease.

Increased sodium concentration in sweat Increased sodium concentration in sweat is a result of impaired chloride ion transport; chloride transport is mediated by the CFTR channel and the gene for this channel is mutated in cystic fibrosis. In the most common CFTR gene mutation, the membrane transporter misfolds, causing it to be retained in the rough endoplasmic reticulum. As a result, chloride is not properly reabsorbed and remains in the lumen of the apocrine sweat gland. Positively charged sodium ions are also pathologically trapped in sweat by the negatively charged chloride ions in order to balance the net electric charge, resulting in hypertonic sweat.

A 47-year-old man with gastroesophageal reflux disease comes to the physician because of severe burning chest pain and belching after meals. He has limited his caffeine intake and has been avoiding food close to bedtime. Esophagogastroduodenoscopy shows erythema and erosions in the distal esophagus. Which of the following is the mechanism of action of the most appropriate drug for this patient? This patient has symptoms consistent with severe GERD, and evidence of reflux-induced esophagitis. A proton pump inhibitor (PPI) would be the appropriate first-line drug therapy for this patient.

Inhibition of ATPase Proton pump inhibitors (PPIs) irreversibly inhibit H+/K+ ATPase in parietal cells of the stomach and thus reduce the ATP-dependent secretion of H+ ions into the gastric lumen. They are considered first-line for patients with severe GERD.

A 55-year-old man with chronic hepatitis B virus infection comes to the physician because of generalized fatigue and a 5.4 kg (12 lb) weight loss over the past 4 months. Physical examination shows hepatomegaly. Laboratory studies show an α-fetoprotein concentration of 380 ng/ml (N < 10 ng/mL). A CT scan of the abdomen with contrast shows a solitary mass in the left lobe of the liver that enhances in the arterial phase. Which of the following is the most likely underlying pathogenesis of this patient's current condition? A solitary hepatic mass in an individual with elevated α-fetoprotein levels is strongly suggestive of hepatocellular carcinoma (HCC). Chronic hepatitis B virus infection is an important risk factor for HCC

Integration of foreign DNA into host genome Integration of hepatitis B virus DNA into the host genome alters the expression of endogenous host genes important for cell growth, proliferation, and differentiation. It also induces chromosomal instability. Although viral DNA integration occurs randomly, it often involves segments of genes encoding for key factors in carcinogenesis, such as p53, pRb, cyclins A and D1, and TGF beta. Therefore, accumulation of genetic damage and altered expression of these regulatory proteins contribute to the development of HCC. Furthermore, continuous expression of certain viral proteins (e.g., HBx protein) that modulate the expression of host genes has also been implicated in the development of HCC.

A 53-year-old man comes to the physician because of a 2-week history of fatigue, generalized itching, and yellowing of the eyes and skin. He underwent a liver transplantation because of acute liver failure following α-amanitin poisoning 1 year ago. Physical examination shows scleral icterus and abdominal distention with shifting dullness. A liver biopsy specimen shows decreased hepatic duct density. Further histological examination of the liver biopsy specimen is most likely to show which of the following findings? This patient with a history of liver transplantation is now exhibiting features of liver failure (jaundice, pruritus, fatigue, ascites). Biopsy shows ductopenia, which indicates vanishing bile duct syndrome.

Interstitial fibrosis Signs and symptoms of liver failure one year after liver transplantation suggests chronic graft rejection, which occurs due to reactions at both the cellular and humoral level in response to donor peptides. The ensuing inflammation leads to interstitial fibrosis, atrophy of the parenchyma, vascular smooth muscle proliferation, and ductopenia (as seen on this patient's biopsy).

Six hours after delivery, a 3100-g (6-lb 13-oz) male newborn has an episode of bilious projectile vomiting. He was born at term to a 21-year-old woman. The pregnancy was complicated by polyhydramnios. The mother smoked a pack of cigarettes daily during the pregnancy. Physical examination shows a distended upper abdomen. An x-ray of the abdomen shows 3 distinct, localized gas collections in the upper abdomen and a gasless distal abdomen. Which of the following is the most likely diagnosis? The 3 localized gas collections ("triple bubble sign") are in 3 separate portions of the GI tract proximal to an obstruction.

Jejunal atresia A distended abdomen and bilious emesis suggest an intestinal obstruction distal to the pylorus. A triple bubble sign on abdominal x-ray confirms the diagnosis of jejunal atresia. This malformation occurs due to a vascular disruption of the mesenteric blood vessels in utero, resulting in necrosis and segmental reabsorption of the intestine. The residual small bowel distal to the atresia may wind around the stalk of the ileocolic artery in spirals, resembling an apple peel (apple peel atresia). Maternal use of vasoconstrictive drugs (e.g., cocaine, MDMA, or cigarettes) increases the risk for this malformation.

A 26-year-old woman, gravida 2, para 2, comes to the physician because of diffuse abdominal pain and intermittent constipation over the past year. She has also had several episodes of nonbloody diarrhea that last up to a week then spontaneously resolve. Symptoms only occur during the day and usually improve after bowel movements. Upon questioning, she reports high levels of stress from taking care of her two toddlers while her husband travels for work. Menses occur at regular 28-day intervals with moderate flow that last for 4 to 5 days and are associated with severe pain. She does not smoke or drink alcohol. She takes naproxen for menstrual pain and an oral contraceptive. Her temperature is 37.6°C (99.7°F), pulse is 94/min, and blood pressure is 115/70 mm Hg. Physical examination shows no abnormalities. Her hemoglobin concentration is 12.8 g/dL, leukocyte count is 5300/mm3, and erythrocyte sedimentation rate is 17 mm/h. IgA antitissue transglutaminase antibodies are negative. Which of the following is the most likely diagnosis? The first-line treatment for this condition is lifestyle changes and dietary adjustments.

Irritable bowel syndrome Irritable bowel syndrome (IBS) is a clinical diagnosis that manifests with recurrent abdominal pain that is associated with ≥ 2 of the following: pain related to defecation, change in stool frequency, and change in stool consistency. Red flag symptoms (e.g., nighttime diarrhea, bloody stools, fever, weight loss) are absent in this condition. IBS symptoms are often triggered by stress, and lab findings are usually normal, as seen in this patient. First-line treatment includes lifestyle modifications (e.g., physical activity, stress management), dietary adjustments (e.g., adequate hydration, high-fiber foods, avoidance of FODMAP foods), and psychotherapy (e.g., cognitive behavioral therapy). Antidiarrheals (e.g., loperamide) may be prescribed for diarrhea-predominant IBS, laxatives (e.g., psyllium, polyethylene glycol, lubiprostone) may be prescribed for constipation-predominant IBS, and antispasmodics (e.g., dicyclomine, hyoscyamine) or tricyclic antidepressants may be prescribed to alleviate cramping/pain.

A 38-year-old woman comes to the physician because of a 1-month history of fatigue and pruritus. Examination of the abdomen shows an enlarged, nontender liver. Serum studies show an alkaline phosphatase level of 200 U/L, aspartate aminotransferase activity of 28 U/L, and alanine aminotransferase activity of 29 U/L. Serum antimitochondrial antibody titers are elevated. A biopsy specimen of this patient's liver is most likely to show which of the following findings? Fatigue, pruritus, hepatomegaly, elevated alkaline phosphatase, and elevated antimitochondrial antibodies (AMA) titers suggest primary biliary cholangitis (formerly primary biliary cirrhosis).

Lymphocytic infiltration of portal areas and periductal granulomas Primary biliary cholangitis (PBC) most commonly affects middle-aged women and is characterized by lymphocytic inflammation of the portal area with periductal granulomatous changes and eventual bile duct destruction. Symptoms include fatigue, hyperpigmentation, and pruritus. PBC is typically diagnosed through elevated serum alkaline phosphatase levels and normal serum total bilirubin levels. Antimitochondrial antibodies (AMAs) are present in 90%-95% of patients. For patients with undetectable serum AMA levels, a liver biopsy is required for diagnosis.

A 44-year-old man comes to the physician because of fatigue and increased straining during defecation for 3 months. During this time, he has lost 5 kg (12 lb) despite no change in appetite. He has a family history of colon cancer in his maternal uncle and maternal grandfather. His mother died of ovarian cancer at the age of 46. Physical examination shows conjunctival pallor. His hemoglobin concentration is 11.2 g/dL, hematocrit is 34%, and mean corpuscular volume is 76 μm3. Colonoscopy shows an exophytic mass in the ascending colon. Pathologic examination of the resected mass shows a poorly differentiated adenocarcinoma. Genetic analysis shows a mutation in the MSH2 gene. Which of the following is the most likely diagnosis? Mutations in the MSH2 DNA mismatch repair gene lead to microsatellite instability. In addition to having an increased risk of colorectal cancer, individuals with this mutation are more likely to develop other cancers such as endometrial, ovarian, and gastric carcinomas.

Lynch syndrome Lynch syndrome is a familial cancer syndrome caused by autosomal dominant mutations in DNA mismatch repair genes (e.g., MSH2, MLH1). Individuals with this syndrome have an ∼ 80% lifetime risk of developing colorectal cancer (typically in the proximal colon). While genetic testing is the test of choice to confirm the diagnosis, the family history-based Amsterdam II criteria can be used to identify individuals who are likely to be affected ("3-2-1 rule": Lynch syndrome-associated cancer in ≥ 3 family members, within 2 generations, and in 1 relative under 50 years of age). This patient fulfills the Amsterdam II criteria.

A 46-year-old woman comes to the physician because of a 3-day history of diarrhea and abdominal pain. She returned from a trip to Egypt 4 weeks ago. Her vital signs are within normal limits. There is mild tenderness in the right lower quadrant. Stool studies show occult blood and unicellular organisms with engulfed erythrocytes. Which of the following is the most appropriate initial pharmacotherapy for this patient? The patient's clinical presentation (bloody diarrhea, abdominal cramps, RLQ tenderness), the history of recent travel to a subtropical region, and the finding of hematophagous trophozoites in stool confirm the diagnosis of intestinal amebiasis, which is caused by Entamoeba histolytica.

Metronidazole Metronidazole is a nitroimidazole antibiotic that is used to treat infections with anaerobic bacteria and certain protozoal infections. It is the drug of choice for the treatment of both symptomatic intestinal amebiasis (amebic dysentery) and extraintestinal amebiasis (e.g., amebic liver abscess). Tinidazole is an effective alternative to metronidazole. Following treatment with metronidazole or tinidazole, all patients with amebiasis should also receive an intraluminal amebicide such as paromomycin or iodoquinol in order to eliminate intestinal carriage of E. histolytica.

A 64-year-old woman with osteoarthritis is brought to the emergency room because of a 2-day history of nausea and vomiting. Over the past few weeks, she has been taking acetaminophen frequently for worsening knee pain. Examination shows scleral icterus and tender hepatomegaly. She appears confused. Serum alanine aminotransferase (ALT) level is 845 U/L, aspartate aminotransferase (AST) is 798 U/L, and alkaline phosphatase is 152 U/L. Which of the following is the most likely underlying mechanism of this patient's liver failure? This patient presents with signs of acute liver failure due to acetaminophen toxicity.

N-acetyl-p-benzoquinoneimine formation N-acetyl-p-benzoquinoneimine (NAPQI) is a toxic breakdown product of acetaminophen. At therapeutic doses, glutathione acts to inhibit toxic NAPQI. However, in cases of acetaminophen overdose, glutathione reserves are depleted and concentrations of NAPQI build up, ultimately leading to acute liver failure, as seen in this patient.

A 54-year-old man comes to the physician because of a 4-month history of yellow discoloration of the skin. He reports a loss of appetite. He lives with his wife and three children in a rural town where he works as a safety inspector at the local nuclear power plant. He is not physically active and his diet consists mostly of donuts and highly caloric processed food. He does not smoke but drinks 5 cans of beer daily. He is 182 cm (6 ft) tall and weighs 101 kg (222 lb); BMI is 30 kg/m2. His vital signs are within normal limits. Physical examination shows jaundice of the skin and scleral icterus. There is a palpable, nontender mass below the right costal margin. Scratch marks are seen on the extremities. Which of the following is the most likely cause of this patient's symptoms? The Springfield Shopper reports that Duff Adequate beer may have been contaminated with uranium for years.

Neoplasia This patient most likely has neoplasia of the gastrointestinal tract that has resulted in biliary system obstruction, as evidenced by his pruritus, loss of appetite, and Courvoisier sign (enlarged gallbladder and painless jaundice). Possible etiologies include pancreatic head carcinoma, cholangiocarcinoma, and hepatocellular carcinoma (HCC). Exposure to ionizing radiation (e.g., from working at a power plant) increases the risk of malignancy. Heavy alcohol consumption is a risk factor for HCC and pancreatic cancer, whereas obesity is a risk factor for pancreatic adenocarcinoma. Diagnostic evaluation should include liver function tests, parameters of cholestasis, and tumor markers (e.g., alpha-fetoprotein, CA 19-9, CEA).

A 42-year-old man comes to the physician because of a 6-week history of intermittent fever, abdominal pain, bloody diarrhea, and sensation of incomplete rectal emptying. He also has had a 4.5-kg (10-lb) weight loss over the past 3 months. Abdominal examination shows diffuse tenderness. Colonoscopy shows circumferential erythematous lesions that extend without interruption from the anal verge to the cecum. A biopsy specimen taken from the rectum shows mucosal and submucosal inflammation with crypt abscesses. This patient is most likely at risk of developing colon cancer with which of the following characteristics? The patient's symptoms and colonoscopy findings suggest ulcerative colitis (UC).

Non-polypoid dysplasia Recurrent cycles of inflammation, which are a common feature of UC, result in the release of inflammatory markers that initially promote cellular hyperplasia and, ultimately, non-polypoid dysplasia. Although reversible, dysplasia can progress to neoplasia and is the most likely cause of development of colorectal carcinoma in patients with chronic inflammatory bowel diseases.

A 59-year-old man with a history of alcoholic cirrhosis is brought to the physician by his wife for a 1-week history of progressive abdominal distension and yellowing of the eyes. For the past month, he has been irritable, had difficulty falling asleep, become clumsy, and fallen frequently. Two months ago he underwent banding for esophageal varices after an episode of vomiting blood. His vital signs are within normal limits. Physical examination shows jaundice, multiple bruises, pedal edema, gynecomastia, loss of pubic hair, and small, firm testes. There are multiple small vascular lesions on his chest and neck that blanch with pressure. His hands are erythematous and warm; there is a flexion contracture of his left 4th finger. A flapping tremor is seen on extending the forearms and wrist. Abdominal examination shows dilated veins over the anterior abdominal wall, the spleen tip is palpated 4 cm below the left costal margin, and there is shifting dullness on percussion. Which of the following pairs of physical examination findings are caused by the same underlying pathophysiology? Most clinical features of cirrhosis are either due to the failure of the liver to perform its normal functions (e.g., altered bilirubin and sex hormone metabolism, decreased synthesis of proteins, inadequate elimination of nitrogenous waste products) or related to increased pressure within the portal venous system

Palmar erythema and gynecomastia Palmar erythema and gynecomastia are both features of hyperestrogenism. Cirrhosis can result in a hyperestrogenic state in which there is an increased estrogen/androgen ratio. This is likely due to increased peripheral conversion of androgens to estrogens and decreased hepatic breakdown of estrogen in men with cirrhosis, as well as increased binding and thus inactivation of free testosterone caused by increased amounts of sex-hormone-binding globulin. Other features of hyperestrogenism seen in this patient are spider angiomata and loss of pubic hair.

A 37-year-old woman comes to the physician because of a 6-month history of weight loss, bloating, and diarrhea. She does not smoke or drink alcohol. Her vital signs are within normal limits. She is 173 cm (5 ft 8 in) tall and weighs 54 kg (120 lb); BMI is 18 kg/m2. Physical examination shows bilateral white spots on the temporal half of the conjunctiva, dry skin, and a hard neck mass in the anterior midline that does not move with swallowing. Urinalysis after a D-xylose meal shows a normal increase in renal D-xylose excretion. Which of the following is most likely to have prevented this patient's weight loss? This patient has xeroderma and keratin deposits on the conjunctivae (Bitot's spots), which are features suggestive of vitamin A deficiency. The physiologic increase in D-xylose excretion seen in this patient indicates that the integrity of the intestinal mucosa is intact.

Pancreatic enzyme replacement This patient's hard, immobile neck mass in the anterior midline is suggestive of Riedel's thyroiditis, which is associated with other IgG4-related systemic diseases. Her xeroderma and Bitot's spots indicate a vitamin A deficiency, and her weight loss, diarrhea, and physiologic increase in urinary D-xylose excretion indicate exocrine pancreatic insufficiency, most likely due to autoimmune pancreatitis. Pancreatic enzyme replacement would be the preventative treatment.

A 46-year-old man comes to the physician with a 1-week history of yellowish discoloration of his eyes, generalized fatigue, and pruritus. He was diagnosed with ulcerative colitis 7 years ago. At the time of diagnosis, a pANCA test was also positive. Physical examination shows scleral icterus and multiple scratch marks on the trunk and extremities. Abdominal examination is unremarkable. Serum studies show a total bilirubin concentration of 3.2 mg/dL, direct bilirubin concentration of 2.5 mg/dL, and alkaline phosphatase level of 450 U/L. Magnetic resonance cholangiopancreatography shows focal areas of intrahepatic bile duct strictures alternating with areas of dilation. Microscopic examination of a liver biopsy specimen is most likely to show which of the following findings? This patient with ulcerative colitis has pruritus, icterus, elevated parameters of cholestasis, and alternating strictures and dilation on MRCP (beading pattern). A well-known cause of cholestatic jaundice in patients with autoimmune diseases, particularly ulcerative colitis, is primary sclerosing cholangitis.

Periductal concentric scarring and fibrosis This patient's symptoms, laboratory, and imaging studies are consistent with the diagnosis of primary sclerosing cholangitis (PSC). In 90% of cases, this condition is associated with inflammatory bowel disease (IBD), usually ulcerative colitis, which this patient has. Histopathological examination of bile ducts would typically show periductal concentric scarring and fibrosis that are consistent with the characteristic onion-skin appearance. PSC, which is classically seen in middle-aged men, increases the risk of development of cancer of the gallbladder and cholangiocarcinoma (10-15% of cases). Perinuclear anti-neutrophil cytoplasmic antibodies (pANCA) are not specific to PSC but can be present in up to 80% of cases.

A 58-year-old woman comes to the physician for a 3-month history of gradual swelling on the right side of her face. She has smoked 1 pack of cigarettes daily for the past 18 years. Physical examination shows a right-sided, movable facial mass that is nontender to palpation. There is a visible protrusion of the right lateral wall of the oropharynx. The patient is able to fully open her mouth. There is no cervical lymphadenopathy. Cranial nerves are intact. An MRI shows a 3.3-cm sharply demarcated, lobulated hyperintense mass arising from the right parotid gland. The lesion is surrounded by a hypointense capsule. A biopsy of the mass shows nests of epithelial cells mixed with a chondromyxoid stroma. Which of the following is the most likely diagnosis? A well-encapsulated mass, in the absence of signs of tumor invasion into the surrounding tissues (e.g., facial nerve palsy, trismus) or lymphatic spread, suggests a benign lesion of the parotid gland.

Pleomorphic adenoma A gradually growing, painless, mobile, and well-encapsulated unilateral mass arising from the parotid gland without facial nerve palsy should raise suspicion for a pleomorphic adenoma, the most common benign tumor of the salivary glands. An admixture of epithelial components and chondromyxoid stroma are pathognomonic for pleomorphic adenomas. Because these benign growths have the potential to undergo malignant transformation, complete resection is recommended.

A 12-year-old girl is brought to the physician because of a 2-hour history of severe epigastric pain, nausea, and vomiting. Her father has a history of similar episodes of abdominal pain and developed diabetes mellitus at the age of 30 years. Abdominal examination shows guarding and rigidity. Ultrasonography of the abdomen shows diffuse enlargement of the pancreas; no gallstones are visualized. Which of the following is the most likely underlying cause of this patient's condition? The patient's severe epigastric pain and ultrasonography showing diffuse enlargement of the pancreas suggest acute pancreatitis, which usually occurs in older patients with a history of gallstones or longstanding alcohol use disorder. The absence of these risk factors, the atypical age of presentation, and the family history of similar episodes suggests there is a hereditary component to this patient's disease.

Premature activation of trypsinogen Premature activation of trypsinogen to trypsin within the pancreas activates the pancreatic digestive enzymes (lipase, amylase, and protease) that cause pancreatic autodigestion and inflammation. This enzymatic pathway is the underlying pathophysiology of acute pancreatitis, regardless of the etiology (e.g., pancreatic ductal blockage in gallstone pancreatitis, pancreatic acinar injury in alcohol-induced pancreatitis). Hereditary pancreatitis, which this patient most likely has, is usually caused by a mutated PRSS1 gene that promotes intrapancreatic trypsinogen activation.

A 71-year-old woman comes to the physician with a 2-month history of fatigue, anorexia, abdominal swelling, shortness of breath, and a 5-kg (11-lb) weight loss. She appears chronically ill. Examination shows jaundice, bilateral temporalis muscle wasting, hepatosplenomegaly, and tense ascites. Ultrasonography of the abdomen shows multiple hepatic masses and enlargement of the portal vein. Which of the following is the most likely cause of these masses? Processed meats are a risk factor for the causative condition.

Processed meats are a risk factor for the causative condition. Metastatic spread of malignant cells from the colon The most common malignant lesion of the liver is metastatic liver disease, which can manifest with muscle wasting (due to cancer cachexia), hepatosplenomegaly, and features of portal hypertension (e.g., enlarged portal vein, ascites). A liver ultrasound typically shows multiple lesions, as seen in this patient. The most common primary tumor in the case of liver metastases is a colorectal carcinoma, which can be caused by a diet rich in processed meats and low in fiber. Other primary tumor sites for hepatic metastases include the lungs and breasts.

An otherwise healthy 50-year-old man comes to the physician because of a 6-month history of increasingly frequent episodes of upper abdominal pain, nausea, vomiting, and diarrhea. He has had a 3.2-kg (7-lb) weight loss during this time. Physical examination shows bilateral pitting pedal edema. An endoscopy shows prominent rugae in the gastric fundus. Biopsy shows parietal cell atrophy. Which of the following is the most likely underlying cause? This patient presents with dyspeptic symptoms (i.e., abdominal pain, nausea, vomiting, and diarrhea), weight loss, and possible hypoproteinemia (as evidenced by bilateral pedal edema). In combination with prominent gastric rugae on endoscopy and atrophic parietal cells on histology, these features are diagnostic of Ménétrier disease.

Proliferation of gastric mucus-producing cells The patient's findings are consistent with Ménétrier disease (protein-losing hypertrophic gastropathy), which is a result of the proliferation of gastric mucus-producing cells. The pathogenesis involves increased signaling of EGFR, which results in proliferation of epithelial cells of the mucous cell compartment.

A 37-year-old man comes to the physician because of a 3-day history of fatigue and yellowish discoloration of his eyes and skin. Physical examination shows mild right upper quadrant abdominal tenderness. The course of different serum parameters over the following 4 months is shown. Which of the following is the most likely explanation for the course of this patient's laboratory findings? Resolved acute hepatitis B infection ( look for the pic online) The depicted natural history of hepatitis B virus infection is observed in 95% of infected adults.

Resolved acute hepatitis B infection The serologic findings represent the natural history of a resolving acute hepatitis B virus infection. After the infection and incubation period, serum findings show HBsAg and HBeAg, followed shortly by the appearance of antibodies against the hepatitis B core antigen (predominantly IgM isotype in the window phase). With resolution of the acute infection, patients remain negative for HBeAg and HBsAg and positive for anti-HBe and anti-HBs antibodies.

A 49-year-old man comes to the physician because of a 6-month history of increasing fatigue and reduced libido. He also complains of joint pain in both of his hands. His vital signs are within normal limits. Physical examination shows tanned skin and small testes. The second and third metacarpophalangeal joints of both hands are tender to palpation and range of motion is limited. The liver is palpated 2 to 3 cm below the right costal margin. Histopathologic examination of a liver biopsy specimen shows intracellular material that stains with Prussian blue. This patient is at greatest risk for developing which of the following complications? Skin hyperpigmentation, testicular atrophy, hepatomegaly, and siderosis with Prussian blue staining are all suggestive of hemochromatosis.

Restrictive cardiomyopathy In patients with untreated or advanced hemochromatosis, the deposition of iron in cardiac tissues and subsequent fibrosis can lead to either a restrictive cardiomyopathy (more typical) or dilated cardiomyopathy (less common, but often reversible).

A previously healthy 47-year-old woman comes to the emergency department because of a 2-week history of fatigue, abdominal distention, and vomiting. She drinks 6 beers daily. Physical examination shows pallor and scleral icterus. A fluid wave and shifting dullness are present on abdominal examination. The intravascular pressure in which of the following vessels is most likely to be increased? Given this patient's heavy alcohol consumption, her symptoms of vomiting, fatigue, ascites, and jaundice are highly suggestive of alcoholic cirrhosis.

Short gastric vein The short gastric veins drain into the splenic vein, which joins the superior mesenteric vein to form the hepatic portal vein. In patients with liver cirrhosis, the increased intraluminal pressure from portal hypertension causes engorgement of these vessels. This can result in gastric varices that can be complicated by life-threatening upper gastrointestinal bleeding.

A previously healthy 49-year-old woman comes to the emergency department because of chest pain that radiates to her back. The pain started 45 minutes ago while she was having lunch. Over the past 3 months, she has frequently had the feeling of food, both liquid and solid, getting "stuck" in her chest while she is eating. The patient's vital signs are within normal limits. An ECG shows a normal sinus rhythm with no ST-segment abnormalities. An esophagogram is shown. Further evaluation is most likely to show which of the following? The esophagogram in this patient with chest pain and dysphagia to solids and liquids shows a corkscrew or rosary bead appearance, which is typical of diffuse esophageal spasm (DES).

Simultaneous multi-peak contractions on manometry Simultaneous multi-peak contractions on esophageal manometry are the characteristic finding of Diffuse esophageal spasm (DES). Repetitive nonperistaltic, nonprogressive contractions impede the progression of solid and liquid foods down the esophagus, which classically leads to dysphagia and/or a squeezing retrosternal chest pain, as seen in this patient. Medical treatment consists of calcium channel blockers, anticholinergics, or nitrates.

A 57-year-old man comes to the physician for a follow-up evaluation of chronic, retrosternal chest pain. The pain is worse at night and after heavy meals. He has taken oral pantoprazole for several months without any relief of his symptoms. Upper endoscopy shows ulcerations in the distal esophagus and a proximally dislocated Z-line. A biopsy of the distal esophagus shows mature columnar epithelium with goblet cells. Which of the following microscopic findings underlies the same pathomechanism as the cellular changes seen in this patient? The biopsy findings in this patient (columnar epithelium with goblet cells) confirm a diagnosis of Barrett esophagus, which is characterized by metaplasia from squamous epithelium to intestinal columnar epithelium as a result of chronic gastric acid exposure.

Squamous epithelium in the bladder The bladder is normally lined by transitional epithelium. Chronic irritation of the bladder wall (e.g., from Schistosoma infection, urinary calculi, or indwelling catheters) can result in a transformation of transitional to squamous epithelium. Cellular transformation due to chronic stress (metaplasia) occurs when stem cells reprogram to replace one mature cell type with another that adapts to the chronic stress. Early metaplastic changes are reversible if exposure to the irritant stops. However, dysplastic changes can occur if chronic irritation persists.

A 3-day-old newborn is brought to the physician because of abdominal distention, inconsolable crying, and 3 episodes of bilious vomiting since the previous evening. He was delivered at home at 40 weeks' gestation by a trained midwife. He has not passed meconium. Physical examination shows abdominal distention, a tight anal sphincter, and an explosive passage of air and feces on removal of the examining finger. Abnormal development of which of the following best explains this patient's condition? This newborn's bilious vomiting, abdominal distention, failure to pass meconium in the first 48 hours of life, tight anal sphincter, and explosive passage of stool on examination are consistent with Hirschsprung disease.

Submucosa and muscularis externa The submucosa normally contains the Meissner (submucous) plexus, while the muscularis externa contains the Auerbach (myenteric) plexus. In Hirschsprung disease, these two plexuses are absent at the anorectal line. Hirschsprung disease is caused by defective migration of plexus ganglion cell precursors (neural crest cells), which ordinarily migrate to the distal colon during fetal development. Without the plexuses, peristalsis is uncoordinated, motility is slow, and relaxation is impeded, resulting in excessive contraction of intestinal muscles, constipation, and obstruction, as seen here. A biopsy of the distal colon showing absent ganglion cells is diagnostic.

Two days after undergoing abdominal surgery for lysis of adhesions, a 52-year-old man has nausea and one episode of bilious vomiting. The patient's nausea is somewhat alleviated in the prone position. The patient has had a 70-kg (154-lb) weight loss since undergoing bariatric surgery 1 year ago. Physical examination shows abdominal distention. Sudden movement of the patient elicits a sloshing sound on auscultation of the abdomen. An upper gastrointestinal series of the abdomen with oral contrast shows no passage of contrast past the third segment of the duodenum. The obstruction in this patient is most likely caused by which of the following structures? The structure in question can compress the part of the duodenum lying between it and the aorta, leading to features of small bowel obstruction such as nausea, bilious vomiting, abdominal distention, and succussion splash (a sloshing sound heard on abdominal auscultation after moving the patient suddenly).

Superior mesenteric artery The superior mesenteric artery passes anterior to the third segment of the duodenum (transverse duodenum), and the aorta is located directly posterior to this part of the intestine. Compression of the duodenum by this artery is called superior mesenteric artery syndrome. The classic presentation includes postprandial pain and bilious emesis following extreme weight loss. Rapid weight loss causes shrinkage of the fat pad between the superior mesenteric artery and the duodenum, leading to compression of the intestinal lumen. Pediatric patients with congenitally abnormal arterial anatomy or those undergoing corrective surgery for scoliosis are also at risk.

A 38-year-old woman comes to the physician because of a 3-month history of moderate abdominal pain that is unresponsive to medication. She has a history of two spontaneous abortions at 11 and 12 weeks' gestation. Ultrasound examination of the abdomen shows normal liver parenchyma, a dilated portal vein, and splenic enlargement. Upper endoscopy shows dilated submucosal veins in the lower esophagus. Further evaluation of this patient is most likely to show which of the following findings? A dilated portal vein, splenic enlargement, and dilated lower esophageal submucosal veins with an ultrasound showing normal liver parenchyma suggests portal hypertension due to portal vein thrombosis.

Thrombocytopenia Thrombocytopenia is a common presentation in patients with an enlarged spleen because enlargement can increase splenic activity (hypersplenism), leading to the rapid clearance of platelets from the blood stream. Splenic enlargement and esophageal varices (collateral circulation) are seen in all forms of portal hypertension. However, unlike post-hepatic and hepatic causes of portal hypertension (e.g., Budd-Chiari syndrome, cirrhosis), pre-hepatic causes of portal hypertension such as splenic or portal vein thrombosis are usually not associated with ascites. Repeated spontaneous miscarriages in patients with thrombophilia, as suggested here by the diagnosis of portal vein thrombosis, is suspicious of antiphospholipid syndrome (APS).

A 79-year-old man with aortic stenosis comes to the emergency room because of worsening fatigue for 5 months. During this time, he has also had intermittent bright red blood mixed in with his stool. He has not had any abdominal pain or weight loss. Physical examination shows pale conjunctivae and a crescendo-decrescendo systolic murmur best heard at the second right intercostal space. The abdomen is soft and non-tender. Laboratory studies show a hemoglobin of 8 g/dL and a mean corpuscular volume of 71 μm3. Colonoscopy shows no abnormalities. Which of the following is the most likely underlying mechanism of this patient's bleeding? This patient has evidence of a chronic, episodic lower GI bleed without mass or diverticula on colonoscopy. His murmur is consistent with aortic stenosis, which is commonly associated with the condition causing his GI bleeding.

Tortuous submucosal blood vessels Tortuous and dilated submucosal blood vessels are characteristic of angiodysplasia, which is associated with aortic stenosis (as seen in this patient). The dysplastic vessels seen in angiodysplasia are typically located in the cecum and ascending colon (in > 75% of cases) and classically cause episodic and self-limiting bleeding. Colonoscopic visualization is challenging and definitive diagnosis usually requires angiography. Other conditions associated with angiodysplasia include von Willebrand disease and end-stage renal disease.

A 67-year-old woman comes to the physician because of a 9-month history of progressive fatigue. Examination shows pallor. Her hemoglobin concentration is 8.9 g/dL, mean corpuscular volume is 75 μm3, and serum ferritin is 6 ng/mL. Test of the stool for occult blood is positive. Colonoscopy shows an irregular, bleeding 3-cm exophytic ulcer in the right colon. Which of the following lesions is the greatest risk factor for this patient's condition? This patient presents with symptomatic iron deficiency anemia and is found to have an irregular, ulcerative growth in the right colon. These findings are highly suggestive of colorectal cancer.

Villous adenomatous polyp Adenomatous polyps (i.e., tubular adenoma, tubulovillous adenoma, or villous adenoma) carry the highest risk of malignant transformation of all types of colonic polyps. A villous adenomatous polyp has the highest risk (∼ 50%) of malignant transformation of the three types of adenomatous polyps.

A 69-year-old man comes to the physician because of progressive difficulty swallowing and a 5-kg (11-lb) weight loss over the past 3 months. He first had trouble swallowing solid foods and then also developed difficulty swallowing liquids over the past week. Endoscopy shows a large mass 3 cm proximal to the esophagogastric junction. Biopsy of the mass shows significant distortion of glandular architecture. Which of the following is the strongest predisposing factor for this patient's condition? Visualization of a mass in the distal third of the esophagus with histopathologic examination that shows distorted glandular architecture confirms a diagnosis of esophageal adenocarcinoma.

Visceral obesity Obesity is an important risk factor for esophageal adenocarcinoma. Although the exact mechanism is unknown, it is hypothesized that obesity increases the risk of GERD, which induces metaplasia (Barrett esophagus) that can progress to adenocarcinoma. In addition, excessive visceral fat increases the risk of inflammation, the production of cytokines and growth factors, and insulin resistance, all of which are risk factors for carcinoma.

A 70-year-old man with a 1-year history of constipation comes to the physician for a follow-up examination. He has bowel movements 1-2 times a week and sometimes has to use his fingers for stool evacuation. Defecation is painful and preceded by a sense of bowel obstruction. Flexible sigmoidoscopy two months prior showed normal results. His current medication is a laxative. Abdominal examination shows no abnormalities. Digital examination of the rectum shows an increased resting and squeezing tone of the anal sphincter. This patient's symptoms are most likely caused by dysfunction of which of the following processes? This patient most likely has primary constipation due to an inability to coordinate relaxation of the pelvic muscles that are involved in defecation.

Voluntary relaxation of the puborectalis muscle The puborectalis muscle, part of the levator ani muscle group, must be voluntarily relaxed to facilitate defecation. This patient most likely has pelvic dyssynergia, which leads to increased sphincter tonus and inability to empty the anal canal. Normally, defecation is enabled by simultaneous voluntary contraction of abdominal muscles and relaxation of the external anal sphincter and puborectalis muscle.

Peutz-Jeghers syndrome

A autosomal dominant, hamartomatous polyposis syndrome characterized by the presence of polyps (typically < 20) throughout the gastrointestinal tract (mainly the jejunum). Associated with mutation of the STK11 gene on chromosome 19p13.3. Manifests with hematochezia, constipation, diarrhea, mucocutaneous hyperpigmentation, and an increased risk of colorectal, ovarian, breast, and pancreatic cancer.

Echinococcosis Hydatidosis, Hydatid disease, Hydatid cyst, Echinocococcal cyst

A helminthic infection caused by cestodes of the genus Echinococcus (e.g., E. granulosus, E. multilocularis). Infection is typically acquired by accidental ingestion of Echinococcus eggs, which are shed in the feces of infected dogs. Infection results in cystic lesions in the liver (75% of cases), lung (15% of cases), and, rarely, other organs.

Vanishing bile duct syndrome Bile duct paucity, Hepatic ductopenia

A histopathologic finding characterized by paucity or absence of bile ducts. Caused by congenital disorders that impair intrahepatic bile duct formation (e.g., congenital hepatic fibrosis, Alagille syndrome, cystic fibrosis, alpha-1 antitrypsin deficiency) and disorders that damage the biliary tree (e.g., primary biliary cholangitis, primary sclerosing cholangitis, chronic graft-versus-host disease, transplant rejection).

Trismus

A limited ability to open the jaw due to tonic spasms of the jaw musculature. Most commonly associated with tetanus. Can also arise secondary to peritonsillar abscess, infection in the jaw muscles, and trigeminal nerve pathology.

A 34-year-old man comes to the physician because of foul-smelling diarrhea, fatigue, and bloating for 6 months. During this time, he has had a 5-kg (11-lb) weight loss without a change in diet. He has type 1 diabetes mellitus that is well-controlled with insulin. Examination shows conjunctival pallor and inflammation of the corners of the mouth. The abdomen is soft, and there is diffuse tenderness to palpation with no guarding or rebound. His hemoglobin concentration is 10.4 g/dL. The patient undergoes upper endoscopy. A photomicrograph of tissue from an intestinal biopsy is shown. Which of the following is most likely to improve this patient's symptoms? This patient's duodenal biopsy shows villous flattening, elongated and hyperplastic crypts of Lieberkuhn, and lymphocytic infiltration of the lamina propria.

Avoidance of certain types of cereal grains Foul-smelling diarrhea, fatigue, bloating, unintentional weight loss, abdominal tenderness, and signs of iron deficiency anemia (low hemoglobin, conjunctival pallor, angular cheilitis) should raise suspicion for celiac disease, for which type 1 diabetes mellitus is a risk factor. The diagnosis is confirmed by the presence of characteristic histology findings together with positive celiac serology. While the disease can occur at any age, the age of onset follows a bimodal distribution; it most often affects infants and adults in the 3rd and 4th decade of life. Treatment consists of a lifelong gluten-free diet, including avoidance of certain types of cereal grains such as wheat, rye, barley, and spelt.

A 56-year-old woman comes to the physician because of a 2-year-history of intermittent upper abdominal pain that occurs a few hours after meals and occasionally wakes her up in the middle of the night. She reports that the pain is relieved with food intake. Physical examination shows no abnormalities. Endoscopy shows a 0.5 x 0.5 cm ulcer on the posterior wall of the duodenal bulb. A biopsy specimen obtained from the edge of the ulcer shows hyperplasia of submucosal glandular structures. Hyperplasia of these cells most likely results in an increase of which of the following? Brunner glands are located in the submucosa of the duodenum and undergo hyperplasia as a protective mechanism in patients with peptic ulcer disease.

Bicarbonate secretion Brunner glands are located in the submucosa of the duodenum and secrete an alkaline, bicarbonate-rich fluid to neutralize stomach acid. The duct openings deliver the secretions into the base of the crypts of Lieberkuhn. Peptic ulcer disease can lead to compensatory hyperplasia of Brunner glands.

A 21-year-old woman comes to the physician because of a 2-month history of fatigue, intermittent abdominal pain, and bulky, foul-smelling diarrhea. She has had a 4-kg (8-lb 12-oz) weight loss during this period despite no changes in appetite. Examination of the abdomen shows no abnormalities. Staining of the stool with Sudan III stain shows a large number of red droplets. Which of the following is the most likely underlying cause of this patient's symptoms? Sudan III staining is used to identify lipids in a sample. Foul-smelling diarrhea with fat in the stool (steatorrhea) indicates fat malabsorption.

Celiac disease Celiac disease (CD) is an inflammatory autoimmune condition caused by hypersensitivity to gliadin. It predominantly affects the duodenum and jejunum and is characterized by villous atrophy, crypt hyperplasia, and intraepithelial lymphocytosis. Patients are often asymptomatic but can present at any age with gastrointestinal symptoms such as crampy abdominal pain, fat malabsorption, and steatorrhea as well as extraintestinal manifestations (e.g., symptoms of fat-soluble vitamin deficiency, anemia, depression, dermatitis herpetiformis). Patients with celiac disease are often positive for IgA anti-tissue transglutaminase, anti-endomysial, and anti-deamidated gliadin peptide antibodies. Celiac disease is most common among individuals of northern European descent and is associated with HLA-DQ2 and HLA-DQ8 haplotypes.

A 63-year-old man with diverticular disease comes to the emergency department because of painless rectal bleeding, dizziness, and lightheadedness for 2 hours. His temperature is 37.6°C (99.6°F), pulse is 115/min, respirations are 24/min, and blood pressure is 86/60 mm Hg. He appears pale. Physical examination shows bright red rectal bleeding. Colonoscopy shows profuse diverticular bleeding; endoscopic hemostasis is performed. After initiating fluid resuscitation, the patient becomes hemodynamically stable. The following day, laboratory studies show: Hemoglobin 8 g/dL Leukocyte count 15,500/mm3 Platelet count 170,000/mm3 Serum Urea nitrogen 60 mg/dL Creatinine 2.1 mg/dL Bilirubin Total 1.2 mg/dL Indirect 0.3 mg/dL Alkaline phosphatase 96 U/L Alanine aminotransferase (ALT, GPT) 2,674 U/L Aspartate aminotransferase (AST, GOT) 2,254 U/L Which cells in the patient's liver were most likely damaged first? This patient's extremely high serum transaminases with relatively normal cholestatic parameters (normal alkaline phosphatase, mildly elevated direct bilirubin), previous hemodynamic instability, and evidence of prerenal acute kidney injury (BUN:Cr > 20:1) raise suspicion for ischemic hepatitis.

Centrilobular hepatocytes The liver is organized microscopically into hepatic lobules (anatomic unit), portal lobules (bile-synthetic functional unit), and hepatic acini (microvascular unit). The hepatic acinus is a diamond-shaped zone between two neighboring central veins (long axis) and two opposing portal triads (short axis). Within the acinus, blood enters through the portal triad (75% deoxygenated blood from the hepatic portal vein; 25% oxygenated blood from the hepatic artery) and drains through the sinusoidal capillaries into the central vein. Hepatic acini can be divided into three zones, which are distinguished based on the degree of oxygenation. Zone 3 contains centrilobular hepatocytes and surrounds the central vein. Because zone 3 receives oxygen-poor blood, it is the zone most sensitive to ischemic injury, making centrilobular hepatocytes most likely to be damaged first.

A 57-year-old man comes to the physician for a follow-up examination. Serum studies show: Aspartate aminotransferase 134 U/L Alanine aminotransferase 152 U/L γ-Glutamyltransferase 83 U/L (N = 5-50) Hepatitis B surface antigen positive A photomicrograph of a specimen obtained on liver biopsy is shown. These biopsy findings are most characteristic of which of the following types of inflammatory reactions? The patient's biopsy shows signs of tissue destruction and repair, with piecemeal necrosis and ground glass hepatocytes. Together with the serum studies positive for hepatitis B surface antigen (HBsAg), elevated AST/ALT, and elevated GGT, this suggests viral hepatitis.

Chronic inflammation The patient's biopsy shows lymphocytic infiltrates, piecemeal necrosis, and ground glass hepatocytes, which are typical of chronic viral hepatitis. Chronic inflammation of any tissue typically shows mononuclear cell infiltration (e.g., lymphocytes, plasma cells, macrophages) and fibrosis may be present simultaneously as well.

A previously healthy 52-year-old woman is brought to the emergency department after sustaining burns over 45% of her body in a house fire. On arrival, she is in acute distress but is fully oriented. Aggressive intravenous fluid resuscitation is begun and the patient is transferred to the intensive care unit of a burn center. 20 hours later, she has several large, tarry black stools and develops hypotension and tachycardia. Despite appropriate lifesaving measures, she dies. Which of the following is the most likely underlying cause of the patient's tarry black stools? This patient's tarry stools, hypotension, and tachycardia prior to her death suggest an acute upper GI bleed that was precipitated by her extensive burn injury.

Decreased gastric blood flow Curling ulcers are a subtype of stress gastritis seen in patients with extensive burns and occur due to hypovolemia and subsequent hypoperfusion of the stomach. The resultant ischemic tissue injury to the stomach epithelium results in an interruption in the normal mucosal barrier of the stomach, and ulcer formation. PPI should be administered in patients with extensive burns to prevent the formation of Curling ulcers.

A 49-year-old man comes to the physician because of a 1-week history of yellowish discoloration of his skin and generalized pruritus. Examination shows jaundice of the skin and scleral icterus. Urinalysis shows an elevated concentration of bilirubin and a low concentration of urobilinogen. Which of the following is the most likely underlying cause of these findings? This patient's bilirubinuria suggests an underlying conjugated hyperbilirubinemia, as unconjugated bilirubin is tightly bound to albumin and cannot be filtered and excreted in the urine.

Defective hepatic bile excretion Normally, unconjugated bilirubin is absorbed by the liver and conjugated with glucuronic acid. It is then secreted via the bile and converted to urobilinogen by intestinal bacteria. In cases of defective hepatic bile excretion (i.e., due to a bile duct stenosis or choledocholithiasis), conjugation continues to take place (i.e., unconjugated bilirubin concentration remains normal) but conjugated bilirubin cannot be secreted. Conjugated bilirubin accumulates in the blood (conjugated hyperbilirubinemia) and is excreted by the kidneys (increased bilirubin in urinalysis).

A 48-year-old woman comes to the physician because of a 6-month history of excessive fatigue and a 1-month history of progressively increasing generalized pruritus. She has hypothyroidism, for which she receives thyroid replacement therapy. Physical examination shows jaundice. The liver is palpated 4 cm below the right costal margin. Serum studies show a direct bilirubin concentration of 2.9 mg/dL, alkaline phosphatase activity of 580 U/L, and increased titers of antimitochondrial antibodies and anti-thyroid peroxidase antibodies. Which of the following is the most likely cause of this patient's condition? Elevated antimitochondrial antibodies and signs of cholestasis (pruritus, jaundice, and increased ALP and bilirubin concentration) in a middle-aged woman suggest primary biliary cholangitis.

Destruction of intrahepatic bile ducts Destruction of intrahepatic bile ducts is the underlying mechanism of primary biliary cholangitis (PBC). As a result, chronic cholestasis with ensuing secondary hepatic damage and liver cirrhosis can develop. While most patients are asymptomatic in the early stages of the disease, affected individuals can develop cholestatic pruritus, as seen here. Antimitochondrial antibodies are found in 95% of patients with PBC. This patient's hypothyroidism is most likely due to Hashimoto thyroiditis (elevated anti-thyroid peroxidase antibodies), which is commonly associated with PBC.

A 67-year-old man comes to the physician because of a 4-month history of fatigue and weight loss. Physical examination shows jaundice. The liver is palpated 3 cm below the right costal margin. Serum studies show an elevated alpha-fetoprotein and a prolonged prothrombin time. Genetic analysis of a liver biopsy specimen shows a G:C to T:A transversion in codon 249 of the gene coding for the TP53 protein in affected cells. Which of the following risk factors is most specific to the patient's condition? Fatigue, weight loss, jaundice, and hepatomegaly with elevated alpha-fetoprotein and impaired coagulation (elevated INR), are consistent with hepatocellular carcinoma (HCC). The G:C → T:A transversion is present in HCC with a specific etiology.

Dietary aflatoxin exposure Signs of hepatocellular carcinoma and G:C → T:A transversion in codon 249 should raise suspicion for a history of aflatoxin B1 ingestion. Aflatoxin is produced by some strains of the mold Aspergillus. A metabolite of aflatoxin B1 interferes with proper DNA transcription of TP53 gene through an inactivating mutation, resulting in the base transversions seen in this case. This mutation can be found in up to 60% of individuals with HCC in areas with high levels of exposure to aflatoxin B1.

A 45-year-old woman comes to the physician because of a 5-month history of recurrent retrosternal chest pain that often wakes her up at night. Physical examination shows no abnormalities. Upper endoscopy shows hyperemia in the distal third of the esophagus. A biopsy specimen from this area shows nonkeratinized stratified squamous epithelium with hyperplasia of the basal cell layer and neutrophilic inflammatory infiltrates. Which of the following is the most likely underlying cause of this patient's findings? This patient's clinical features and endoscopy and biopsy findings suggest gastroesophageal reflux disease (GERD).

Dysfunction of the gastroesophageal junction This patient's history of intermittent retrosternal chest pain that worsens at night, in conjunction with the hyperemia of the distal third of the esophagus and the biopsy showing nonkeratinized stratified squamous epithelium with hyperplasia of the basal cell layer and neutrophilic inflammatory infiltrates, is consistent with GERD. Dysfunction of the gastroesophageal junction allows stomach contents to flow back into the esophagus, causing inflammation of the esophageal epithelium. Risk factors for GERD include dysfunction of the lower esophageal sphincter due to anatomical disruption (e.g., due to hiatal hernia), decreased tension (e.g., due to smoking), and transient relaxation (e.g., due to obesity).

A 51-year-old man comes to the physician for follow-up evaluation. Nine months ago, he was diagnosed with acute viral hepatitis B infection. Physical examination shows no abnormalities. Serum studies show increased hepatic transaminase activity and a hepatitis B viral DNA load of 4286 IU/mL. Which of the following sets of findings is most likely in this patient? HBeAg Anti-HBs Anti-HBc IgG Anti-HBc IgM A Negative positive positive negative B Negative negative positive negative C Negative positive negative negative D Negative negative negative positive E Positive negative positive negative F Positive negative negative positive This patient who previously had acute hepatitis B infection and currently presents with increased hepatic transaminase activity and an HBV DNA load of 4286 U/L most likely has active chronic hepatitis B infection.

E Up to 10% of patients who develop acute hepatitis B infection will develop chronic hepatitis B infection, which is characterized by the persistence of HBsAg and HBV DNA for more than 6 months after the initial infection, and the absence of anti-HBs. IgG anti-HBc may be present or absent with chronic hepatitis B infection. Chronic hepatitis B infection can be classified as "active" (marked viral replication characterized by increased HBeAg) or "inactive" (low or no viral replication characterized by an absence of serum HBeAg). Patients with active chronic hepatitis B would have an HBV DNA load > 2000 IU/L and/or increased hepatic transaminase levels.

A 4-year-old girl is brought to the emergency department by her father for the evaluation of abdominal pain for 1 hour after drinking a bottle of rust remover. The father reports that she vomited once on the way to the hospital and that her vomit was not bloody. The patient has pain with swallowing. She appears uncomfortable. Oral examination shows mild erythema of the epiglottis and heavy salivation. Which of the following is the most likely long-term complication in this patient? This patient's symptoms (abdominal pain, nausea, vomiting, odynophagia, and heavy salivation) began shortly after the ingestion of rust remover, suggesting caustic liquid poisoning.

Esophageal strictures The most common long-term complication of caustic liquid ingestion is the development of esophageal strictures. Most strictures develop approx. 2 months after the initial injury. Esophageal dysmotility caused by strictures and neuromuscular damage can lead to severe dysphagia. Other complications of caustic liquid ingestion include esophageal ulcers and esophageal cancer.

A 15-month-old girl is brought to the physician by her mother for grossly bloody diarrhea for 2 days. The girl has had a few episodes of blood-tinged stools over the past 3 weeks. She has not had pain, nausea, or vomiting. She is at the 55th percentile for height and 55th percentile for weight. Examination shows conjunctival pallor. The abdomen is soft and nontender. There is a small amount of dark red blood in the diaper. Her hemoglobin concentration is 9.5 g/dL, mean corpuscular volume is 68 μm3, and platelet count is 300,000/mm3. Further evaluation is most likely to show which of the following findings? This young girl presents with painless lower gastrointestinal bleeding and resultant microcytic anemia (likely due to iron deficiency from chronic blood loss), which is the characteristic presentation of Meckel diverticulum.

Ectopic gastric mucosa on Technetium-99m pertechnetate scan Technetium-99m pertechnetate scan showing ectopic gastric mucosa in the small bowel is diagnostic of Meckel diverticulum in children. Meckel diverticulum is a true diverticulum that may contain ectopic gastric mucosa and/or pancreatic tissue. Acid-secretion by gastric mucosa within the diverticulum results in ileal ulceration and subsequent bleeding. Consequently, chronic blood loss can lead to iron deficiency anemia, and thus a low MCV, as seen in this patient.

A 61-year-old man comes to the physician because of fatigue and a 5-kg (11-lb) weight loss over the past 6 months. He experimented with intravenous drugs during his 20s and has hepatitis C. His father died of colon cancer. He has smoked one pack of cigarettes daily for 35 years. Physical examination shows scleral icterus and several telangiectasias on the abdomen. The liver is firm and nodular. Laboratory studies show: Hemoglobin 10.9 g/dL Mean corpuscular volume 88 μm3 Leukocyte count 10,400/mm3 Platelet count 260,000/mm3 Ultrasonography of the liver is shown. Which of the following additional findings is most likely? This patient presents with signs of liver cirrhosis (scleral icterus, telangiectasias, nodular liver surface); ultrasonography shows a solitary, solid, isoechoic hepatic lesion. A history of hepatitis C is a risk factor for this patient's condition.

Elevated α-fetoprotein Both hepatitis infections and liver cirrhosis are risk factors for the development of hepatocellular carcinoma, which is usually asymptomatic except for features of the underlying disease. Diagnostic features include detection of a single hepatic lesion on ultrasound or CT and elevated levels of serum alpha-fetoprotein (AFP). Further symptoms may occur in advanced disease stages, including weight loss, cachexia, ascites, and jaundice.

A 36-year-old woman comes to the physician for difficulty swallowing and a burning sensation in her chest for the past 2 years. She has increased her fluid consumption to help with swallowing when eating. She reports having used both over-the-counter antacids and pantoprazole for 2 months without any relief of symptoms. She uses a daily low-dose inhaled corticosteroid and an albuterol inhaler as needed for allergic asthma. She is 170 cm (5 ft 6 in) tall and weighs 75 kg (165 lbs); her BMI is 26 kg/m2. Her vital signs are within normal limits. Physical examination shows no abnormalities. Findings of an upper endoscopy are shown. Which of the following is the most likely diagnosis? This patient's upper endoscopy shows white, circumferential mucosal lesions (trachealization of the esophagus), mucosa fragility, and linear, longitudinal furrows.

Eosinophilic esophagitis Dysphagia and heartburn refractory to proton pump inhibitors in an individual with a history of atopy are indicative of eosinophilic esophagitis (EoE). A diagnosis of EoE is further supported by the findings on this patient's endoscopy (trachealization of the esophagus, longitudinal furrows, friable mucosa). Individuals with EoE may also present with food impaction, reflux, and vomiting. An endoscopic esophageal biopsy will typically show an eosinophilic infiltrate.

A 30-year-old man is brought to the emergency department because of severe shortness of breath, fever, and cough for 5 days. He says he has smoked one pack of cigarettes daily for 12 years and occasionally uses cocaine on weekends. After intubation in the emergency department for worsening tachypnea and hypoxia, he is admitted to the intensive care unit for management of pneumococcal sepsis. One week later, the patient develops hematemesis. Abdominal examination shows epigastric tenderness; there is no guarding or rebound tenderness. Urgent endoscopy shows multiple shallow hemorrhagic lesions, predominantly in the gastric fundus and greater curvature. Biopsies show patchy epithelial defects that do not extend beyond the muscularis mucosa. Which of the following is the most likely diagnosis? The lesions do not extend into the submucosa, which indicates that they are very superficial.

Erosive gastropathy This patient's history of epigastric pain and hematemesis after pneumococcal sepsis is suggestive of acute erosive gastropathy. Erosions are more superficial than ulcers, which involve damage to the gastric mucosa extending beyond the muscularis mucosa layer into the submucosa. Acute erosive gastropathy is characterized by ischemia of the mucosa due to hypovolemia (e.g., sepsis) and exposure to injurious substances (e.g., smoking, cocaine use). In acute erosive gastropathy, hemorrhage and loss of the superficial epithelium occur, further compromising the normal protective barrier of the stomach and allowing acid to penetrate through the mucosa, which directly damages the vasculature. This damage exacerbates the mucosal ischemia, causing the release of inflammatory mediators and leading to the erosions seen here.

A 64-year-old woman is brought to the emergency department because of a 2-hour history of nausea, vomiting, and retrosternal pain that radiates to the back. Abdominal examination shows tenderness to palpation in the epigastric area. A CT scan of the patient's chest is shown. Which of the following is the most likely diagnosis? The CT scan shows extraluminal gas in the mediastinum (i.e., pneumomediastinum).

Esophageal rupture Boerhaave syndrome is most commonly caused by excessive vomiting and retching (e.g., as a result of high alcohol intake). Increased intrathoracic pressure results in elevated intraesophageal pressure that can lead to full-thickness rupture of the esophageal wall. Patients typically have a history of vomiting and develop severe, retrosternal pain that radiates to the back. Chest auscultation often reveals a crackling or crunching sound (Hamman sign) due to the presence of air in soft tissue spaces. This patient's history, symptoms, and chest CT scan that shows extraluminal gas in the mediastinum (i.e., pneumomediastinum) are consistent with a diagnosis of full-thickness esophageal rupture.

A 22-day-old male newborn is brought to the physician because of poor feeding, lethargy, and an abdominal protrusion. The child was delivered at home and has not yet been evaluated by a physician. The mother has not had routine prenatal care. The patient is at the 25th percentile for height, 50th percentile for weight, and 95th percentile for head circumference. Physical examination shows scleral icterus, pale facies, and an enlarged tongue. The abdomen is distended and there is a protruding mass at the abdominal midline. The skin covering the protrusion appears normal. When the newborn cries during the examination, the mass enlarges but is easily reducible. Which of the following is the most likely cause of the abdominal protrusion? This patient has features consistent with congenital hypothyroidism (poor feeding, lethargy, prolonged newborn jaundice, pale facies, increased head circumference, abdominal distention, and macroglossia). His midline abdominal protrusion is a congenital umbilical hernia, a common feature of congenital hypothyroidism.

Failed spontaneous closure of the umbilical ring Failure of the umbilical ring to close during fetal development causes congenital umbilical hernia. The midgut develops outside the abdominal cavity until the second trimester, when it physiologically herniates back into the abdomen. If the umbilical ring fails to close or the fascia in this region is underdeveloped, abdominal content may bulge through the umbilicus. Umbilical hernias are more common in children with chromosomal abnormalities (e.g., Down syndrome, Edwards syndrome) or congenital hypothyroidism, as seen here. Most congenital umbilical hernias resolve spontaneously by 5 years of age.

A 65-year-old man comes to the physician because of abdominal pain and bloody, mucoid diarrhea for 3 days. He has been taking over-the-counter supplements for constipation over the past 6 months. He was diagnosed with type 2 diabetes mellitus 15 years ago. He has smoked one pack of cigarettes daily for 35 years. His current medications include metformin. His temperature is 38.4°C (101.1°F), pulse is 92/min, and blood pressure is 134/82 mm Hg. Examination of the abdomen shows no masses. Palpation of the left lower abdomen elicits tenderness. A CT scan of the abdomen is shown. Which of the following is the most likely underlying cause of the patient's condition? The patient's CT scan shows segmental thickening and enhancement of the colonic wall with air-filled pouches in the surrounding segments.

Focal weakness of the colonic muscularis layer Focal weakness of the colonic muscularis layer, typically at points where the vasa recta traverse the intestinal wall, allows for the formation of protrusions of mucosal and submucosal layers in response to increased intraluminal pressure (e.g., from chronic constipation). These protrusions (i.e., diverticula) are referred to as "pseudodiverticula" because not all layers of the intestinal wall are affected. Diverticulosis is the asymptomatic presence of these protrusions (identified as air-filled pouches on CT). Diverticulitis is an acute inflammation of diverticula that typically manifests with bowel wall thickening on CT, fever, left-sided lower abdominal pain, and bloody, mucoid diarrhea, all of which are seen in this patient. Risk factors for diverticular disease include diet (low in fiber, high in red meat and fat), BMI ≥ 25 kg/m2, lack of physical activity, and smoking (≥ 40 pack-years).

A 34-year-old woman comes to the emergency department because of a 2-hour history of abdominal pain, nausea, and vomiting that began an hour after she finished lunch. Examination shows abdominal guarding and rigidity; bowel sounds are reduced. Magnetic resonance cholangiopancreatography shows the dorsal pancreatic duct draining into the minor papilla and a separate smaller duct draining into the major papilla. The spleen is located anterior to the left kidney. A disruption of which of the following embryological processes is the most likely cause of this patient's imaging findings? The dorsal pancreatic duct normally drains into the major duodenal papilla.

Fusion of the pancreatic buds Fusion of the dorsal and ventral pancreatic buds normally occurs during the 8th week of embryological development. Subsequently, the distal portion of the dorsal pancreatic duct fuses with the ventral pancreatic duct, which drains into the major duodenal papilla via the Ampulla of Vater. The proximal portion of the dorsal pancreatic duct would degenerate or persist as an accessory pancreatic duct that drains into the minor duodenal papilla. If the pancreatic buds fail to fuse (pancreas divisum), the dorsal and ventral pancreatic duct would drain separately into the duodenum with the dorsal pancreatic duct draining via the minor duodenal papilla, as seen on this patient's MRCP. Most individuals with pancreas divisum are asymptomatic, but a small percentage will present with chronic abdominal pain or pancreatitis, as is the case here.

An otherwise healthy 56-year-old woman comes to the physician because of a 3-year history of intermittent upper abdominal pain. She has had no nausea, vomiting, or change in weight. Physical examination shows no abnormalities. Laboratory studies are within normal limits. Abdominal ultrasonography shows a hyperechogenic rim-like calcification of the gallbladder wall. This patient is at increased risk for which of the following conditions? The imaging findings in this patient are pathognomonic for porcelain gallbladder.

Gallbladder carcinoma Porcelain gallbladder is a calcification of the gallbladder caused by chronic inflammation as a result of cholecystitis. It is often identified incidentally during abdominal imaging, although it may present with right upper quadrant pain, as in this patient. Porcelain gallbladder causes a significant risk of developing gallbladder adenocarcinoma. Therefore, the recommended treatment is cholecystectomy.

A 71-year-old woman comes to the physician because of an 8-month history of fatigue. Laboratory studies show a hemoglobin concentration of 13.3 g/dL, a serum creatinine concentration of 0.9 mg/dL, and a serum alkaline phosphatase concentration of 130 U/L. Laboratory evaluation of which of the following parameters would be most helpful in determining the cause of this patient's symptoms? The patient has nonspecific symptoms and an elevation of serum alkaline phosphatase that may be associated with liver disease or bone pathology. To determine the underlying disease a specific laboratory marker is needed.

Gamma-glutamyl transpeptidase In a patient with increased ALP, gamma-glutamyl transpeptidase (GGT) would be the most appropriate test to differentiate a disease of the hepatobiliary system from a disease of the bone. Simultaneous elevation in both ALP and GGT would signify a liver or biliary tract pathology. If ALP is elevated as a result of increased bone turnover, GGT levels would be normal. Increased GGT with normal ALP is characteristic of heavy alcohol use.

A 59-year-old woman comes to the physician because of a 1-year history of nausea and chronic abdominal pain that is worse after eating. She has Hashimoto thyroiditis. She does not smoke or drink alcohol. A biopsy specimen of the corpus of the stomach shows destruction of the upper glandular layer of the gastric mucosa and G-cell hyperplasia. This patient is at greatest risk for which of the following conditions? The patient has chronic indigestion and gastric biopsy has revealed corporal parietal cell destruction (upper glandular layer). In a patient with a history of autoimmune disease (Hashimoto thyroiditis), this constellation of findings is consistent with autoimmune atrophic gastritis.

Gastric adenocarcinoma Autoimmune atrophic gastritis is a type of chronic gastritis (type A gastritis) that can lead to intestinal metaplasia, which increases the risk of gastric adenocarcinoma. Patients with autoimmune atrophic gastritis are also at increased risk for carcinoid tumor because of the presence of autoantibodies against parietal cells, which normally produce gastric acid. The resulting achlorhydria stimulates the release of gastrin and antral G-cell hyperplasia, resulting in hypergastrinemia. Hypergastrinemia, in turn, is associated with enterochromaffin-like cell hyperplasia, which may progress to gastric carcinoid tumor.

An otherwise healthy 56-year-old man comes to the physician for a 2-year history of recurrent upper abdominal pain and fullness that worsens after meals. Urea breath test is positive. An endoscopy shows diffuse mucosal atrophy and patchy erythema, but no ulcer. A biopsy from which of the following areas is most likely to yield an accurate diagnosis? This patient with recurrent upper abdominal pain and fullness after meals (dyspepsia), positive urea breath test, and upper endoscopy findings of diffuse mucosal atrophy and patchy erythema, likely has atrophic gastritis caused by Helicobacter pylori.

Gastric antrum Chronic atrophic gastritis due to H. pylori infection begins in the gastric antrum (pyloric antrum) and then spreads to other regions such as the fundus and body. Biopsies from the gastric antrum are therefore most likely to yield an accurate diagnosis because atrophic changes and H. pylori infection load is greatest at this site. Gastric MALToma is a potential complication of H. pylori infection. In contrast to H. pylori-induced atrophic gastritis, the gastric fundus and body are primarily affected in autoimmune atrophic gastritis, which can present similarly but the urea breath test would be negative and other features suggestive of autoimmune disease (e.g., pernicious anemia, vitiligo, autoimmune thyroiditis) would usually be present. Patients with atrophic gastritis have decreased gastric acid secretion (hypochlorhydria) with a secondary increase in gastrin levels.

A 49-year-old man comes to the physician because of a 1-week history of diarrhea and abdominal bloating. His stools are bulky, foul-smelling, and difficult to flush. Over the past 6 months, he has had recurrent dull epigastric pain that is triggered by meals and lasts for a few days. He drinks 6 to 8 beers daily. Abdominal examination shows mild epigastric tenderness with no rebound or guarding. A CT scan of the abdomen is shown. The structure indicated by the arrows is most likely lined by which of the following? The large cystic lesion found on CT scan of this patient with a history of chronic alcohol abuse, steatorrhea (bulky, foul-smelling, difficult-to-flush stools), and recurrent epigastric pain is most likely a pancreatic pseudocyst that has formed as a result of chronic pancreatitis.

Granulation tissue Granulation tissue encapsulates pancreatic pseudocysts, retaining enzyme-rich pancreatic fluids. These pseudocysts lack proper epithelial lining (in contrast to true cysts) and are instead walled off by adjacent intraperitoneal structures (e.g., stomach or omentum). Such lesions are due to a disruption of the pancreatic ductal system and extravasation of fluids, usually as a result of acute or chronic pancreatitis. Most pseudocysts remain asymptomatic but large lesions can cause mass effect, resulting in abdominal pain, biliary obstruction, and potentially gastrointestinal bleeds if nearby blood vessels are involved.

A 23-year-old woman comes to the physician because of a 2-month history of diarrhea, flatulence, and fatigue. She reports having 3-5 episodes of loose stools daily that have an oily appearance. The symptoms are worse after eating. She also complains of an itchy rash on her elbows and knees. A photograph of the rash is shown. Further evaluation of this patient is most likely to show which of the following findings? This woman's rash is consistent with dermatitis herpetiformis and her symptoms of malabsorption (e.g., fatty diarrhea, flatulence, fatigue) are suggestive of celiac disease. Macrocytic, hypochromic red blood cells Malabsorptive conditions that cause macrocytic,

HLA-DQ2 serotype HLA-DQ2 is a cell surface receptor of antigen-presenting cells associated with 90-95% of cases of celiac disease. Expression of HLA-DQ2 is thought to predispose patients to celiac disease due to increased binding of gluten peptides to this HLA subtype, triggering a T-cell mediated autoimmune reaction and inflammation of the intestinal epithelium. Dermatitis herpetiformis is associated with celiac disease and is caused by deposition of IgA and complement C3 in dermal papillae of the dermis.

A previously healthy 5-year-old girl is brought to the emergency department by her parents because of a severe headache, nausea, and vomiting for 6 hours. Last week she had fever, myalgias, and a sore throat for several days that resolved with over-the-counter medication. She is oriented only to person. Examination shows bilateral optic disc swelling. Serum studies show: Glucose 61 mg/dL Aspartate aminotransferase (AST) 198 U/L Alanine aminotransferase (ALT)166 U/L Prothrombin time 18 sec Which of the following is the most likely cause of this patient's symptoms? The patient was likely given over-the-counter aspirin for her flu-like symptoms, which should be avoided in pediatric patients because of a rare, life-threatening adverse effect.

Hepatic mitochondrial injury Reye syndrome is caused by hepatic mitochondrial injury that can develop in children 3-5 days after aspirin treatment of a febrile viral illness (most commonly with VZV or influenza B). It initially presents as hepatic encephalopathy that manifests with profuse vomiting (due to ↑ ICP) and lethargy, which progresses to delirium, seizures, and coma. Other typical findings include hepatomegaly, elevated AST and ALT, hypoglycemia, elevated INR, and metabolic acidosis. Liver biopsy is only performed in doubtful cases and shows microvesicular hepatic steatosis (fatty degenerative liver).

Two weeks after returning from vacation in Mexico, a 21-year-old man comes to the emergency department because of malaise, nausea, vomiting, fever, and abdominal pain. He has no history of serious illness and takes no medications. Physical examination shows scleral icterus and right upper quadrant tenderness. The liver is palpated 1.5 cm below the right costal margin. A biopsy specimen of this patient's liver would most likely show which of the following findings? Acute onset of nausea, vomiting, fever, scleral icterus, right upper quadrant abdominal pain, and hepatomegaly following recent travel to Mexico are suggestive of acute hepatitis A infection.

Hepatocyte swelling and bridging necrosis Ballooning degeneration (swelling of hepatocytes due to depletion of ATP) and bridging necrosis (confluent necrosis spanning adjacent lobules) are histopathological findings seen in acute viral hepatitis. Other typical findings include Councilman bodies (reflecting shrunken hepatocytes that underwent apoptosis) and periportal infiltration with helper T cells, B cells, and plasma cells. While the virus has a minimally cytotoxic effect upon cells, the immunological response is the primary method of hepatocyte injury. Hepatitis A infection can manifest with acute symptoms or be asymptomatic, but it does not become chronic, unlike hepatitis B and hepatitis C infections.

A 31-year-old woman is brought to the emergency department with fever, right upper quadrant pain, and myalgia. Her boyfriend says she recently returned from a trip to Southeast Asia. She appears ill and is lethargic. Her temperature is 39°C (102.2°F). Physical examination shows jaundice and tender hepatomegaly. Laboratory studies show the presence of anti-hepatitis A IgM antibodies. A liver biopsy performed at this time would most likely show which of the following histopathological findings? This patient has acute hepatitis A and biopsy is likely to show findings consistent with apoptosis.

Hepatocytes with shrunken, eosinophilic cytoplasm and pyknotic nuclei Hepatocytes with shrunken, eosinophilic cytoplasm and pyknotic nuclei are the histopathological characteristics of Councilman bodies. Councilman bodies are remnants of apoptotic hepatocytes and can be found in patients with acute hepatitis A. Apoptosis of virus-infected cells, such as HAV-infected hepatocytes, is induced by the release of granzyme B and perforin from cytotoxic T cells (the extrinsic pathway of apoptosis). Other histopathological signs in patients with hepatitis A include ballooning degeneration (swelling of hepatocytes) and bridging necrosis (confluent necrosis spanning adjacent lobules).

A 31-year-old woman is brought to the emergency department with fever, right upper quadrant pain, and myalgia. Her boyfriend says she recently returned from a trip to Southeast Asia. She appears ill and is lethargic. Her temperature is 39°C (102.2°F). Physical examination shows jaundice and tender hepatomegaly. Laboratory studies show the presence of anti-hepatitis A IgM antibodies. A liver biopsy performed at this time would most likely show which of the following histopathological findings? This patient has acute hepatitis A and biopsy is likely to show findings consistent with apoptosis.

Hepatocytes with shrunken, eosinophilic cytoplasm and pyknotic nuclei Hepatocytes with shrunken, eosinophilic cytoplasm and pyknotic nuclei are the histopathological characteristics of Councilman bodies. Councilman bodies are remnants of apoptotic hepatocytes and can be found in patients with acute hepatitis A. Apoptosis of virus-infected cells, such as HAV-infected hepatocytes, is induced by the release of granzyme B and perforin from cytotoxic T cells (the extrinsic pathway of apoptosis). Other histopathological signs in patients with hepatitis A include ballooning degeneration (swelling of hepatocytes) and bridging necrosis (confluent necrosis spanning adjacent lobules). Acellular debris surrounded by lymphocytes and macrophages 26%

A 43-year-old woman comes to the physician because of a 2-week history of malaise, nausea, and a 3-kg (6.6-lb) weight loss. She has been drinking eight to ten alcoholic beverages daily for the past 20 years. Her temperature is 37.8°C (100.0°F), and pulse is 105/min. Examination shows jaundice and hepatosplenomegaly. A photomicrograph of a section of a biopsy specimen of the liver is shown. Which of the following mechanisms best explains the findings shown? This patient has a history of chronic alcoholism and presents with signs and symptoms of hepatic injury. The central hepatocyte in the photomicrograph shows ballooning degeneration, as well as a twisted rope-like cytoplasmic inclusion, called a Mallory body; these are both findings that are strongly associated with steatohepatitis.

Increased glycerol 3-phosphate formation Increased glycerol 3-phosphate formation from dihydroxyacetone phosphate (DHAP) tends to occur when there is a high concentration of reduced nicotinamide adenine dinucleotide (NADH), which is present in individuals who engage in heavy drinking. In this case, massive hepatic degradation of ethanol to acetyl-CoA causes NADH excess. An increase in both glycerol 3-phosphate concentration and availability of fatty acids causes increased triglyceride synthesis, leading to the development of steatohepatitis.

A 71-year-old woman with type 2 diabetes mellitus and hypertension comes to the emergency department because of a 3-day history of intermittent abdominal pain, vomiting, and obstipation. She has had multiple episodes of upper abdominal pain over the past year. She has smoked 1 pack of cigarettes daily for the past 30 years. Physical examination shows a distended abdomen with diffuse tenderness and high-pitched bowel sounds. An x-ray of the abdomen shows a dilated bowel, multiple air-fluid levels, and gas in the biliary tree. Which of the following is the most likely cause of this patient's condition? This patient with a long-standing history of episodic upper abdominal pain now presents with features of mechanical obstruction (abdominal distention, vomiting, obstipation, high-pitched bowel sounds, and dilated bowel loops with multiple air-fluid levels) with pneumobilia on abdominal x-ray.

Inflammation of the gallbladder wall Chronic inflammation of the gallbladder wall can lead to the formation of an abnormal connection between the gallbladder and the intestinal tract (i.e., a cholecystoenteric fistula), which provides a conduit for the passage of gallstones. A gallstone can subsequently become lodged in the distal ileum, the section of the intestine with the narrowest lumen, causing gallstone ileus. An abdominal x-ray will show pneumobilia, indicating that intra-intestinal air has moved through to the fistula into the biliary tree.

A 75-year-old man comes to the physician because of a 2-month history of intermittent bright red blood in his stool, progressive fatigue, and a 5-kg (11-lb) weight loss. He appears thin and fatigued. Physical examination shows conjunctival pallor. Hemoglobin concentration is 7.5 g/dL and MCV is 77 μm3. Results of fecal occult blood testing are positive. A colonoscopy shows a large, friable mass in the anal canal proximal to the pectinate line. Primary metastasis to which of the following lymph nodes is most likely in this patient? These lymph nodes also receive lymphatics from the gluteal region, inferior pelvic viscera (including rectum, prostate, and part of the cervix), and the perineum (including membranous urethra and spongy urethra).

Internal iliac The internal iliac lymph nodes receive lymphatic drainage from the upper part of the anal canal, proximal to the pectinate line. The internal iliac lymph nodes would therefore be the most likely site of lymphatic metastasis in this patient with a mass in the proximal anal canal.

A 50-year-old man comes to the physician because of an 8-month history of intermittent watery diarrhea and abdominal pain. He has had a 12-kg (26-lb) weight loss during this period. He has also had episodic pain of the ankle, wrist, and knee joints during the past 5 years. An endoscopy with small bowel biopsy is performed. Histopathologic examination of a tissue specimen shows foamy macrophages in the lamina propria with periodic acid-Schiff (PAS)-positive inclusions. Further evaluation is most likely to show which of the following? Chronic polyarthritis that precedes the development of diarrhea, abdominal pain, and weight loss raises suspicion for Whipple disease. Small bowel biopsy showing PAS-positive, foamy macrophages in the lamina propria confirms the diagnosis.

Intracellular, gram-positive bacilli The causative organism in Whipple disease is Tropheryma whipplei, an intracellular, gram-positive bacillus. Frequently missed in staining, the histological hallmark of classical Whipple disease is periodic acid-Schiff (PAS) reactive vacuoles in macrophages found within the lamina propria of the small bowel, as seen in this patient. Whipple disease is lethal if left untreated. Intravenous antibiotics (ceftriaxone or penicillin plus streptomycin) for 14 days is the recommended induction therapy for Whipple disease, followed by maintenance therapy with trimethoprim-sulfamethoxazole (TMP-SMX) for 1 year.

A 45-year-old woman comes to the physician because of a 3-month history of mild right upper abdominal pain. She has not had any fevers, chills, or weight loss. There is no personal or family history of serious illness. Medications include transdermal estrogen, which she recently started taking for symptoms related to menopause. Abdominal examination shows no abnormalities. Ultrasonography of the liver shows a well-demarcated, homogeneous, hyperechoic mass surrounded by normal liver tissue. A biopsy of the lesion would put this patient at greatest risk for which of the following complications? The ultrasound findings in this patient are consistent with cavernous hemangioma, the most common type of benign liver tumor.

Intraperitoneal hemorrhage Hepatic hemangioma is a benign tumor composed of cavernous vascular spaces of variable size that are lined by flat endothelial cells. They are most common in women between the ages of 30-50 years old and may become enlarged during hormonal therapy. Most hepatic hemangiomas are asymptomatic and diagnosed incidentally on imaging, appearing hyperechogenic (due to increased vascularity), well-demarcated, and homogeneous. Being a highly vascular tumor, biopsy represents a high risk of severe hemorrhage and is not recommended unless the diagnosis is uncertain.

A 62-year-old man comes to the physician because of progressive fatigue and dyspnea on exertion for 3 months. During this time, he has also had increased straining during defecation and a 10-kg (22-lb) weight loss. He has no personal or family history of serious medical illness. Physical examination shows conjunctival pallor. Laboratory studies show microcytic anemia. Test of the stool for occult blood is positive. Colonoscopy shows an exophytic mass in the ascending colon. Pathologic examination of the mass shows a well-differentiated adenocarcinoma. A gain-of-function mutation in which of the following genes is most likely involved in the pathogenesis of this patient's condition? This patient has iron deficiency anemia (progressive fatigue, dyspnea, conjunctival pallor, microcytic anemia, positive stool occult blood) secondary to a well-differentiated, sporadic, right-sided colorectal carcinoma. Most sporadic cases of colorectal carcinoma progress from adenoma to carcinoma due to chromosomal instability.

KRAS Most sporadic cases of colorectal carcinoma arise from the chromosomal instability pathway (i.e., the adenoma-carcinoma sequence). In this pathway, a loss-of-function mutation of the tumor suppressor gene APC results in decreased intercellular adhesion and increased epithelial proliferation. Next, a gain-of-function mutation in the KRAS proto-oncogene results in unregulated cellular signaling and cellular proliferation, which predisposes to adenoma formation. Finally, loss-of-function mutations of additional tumor suppressor genes (e.g., TP53, DCC) results in malignant transformation of an adenoma to a carcinoma.

A 55-year-old woman comes to the physician because of a 2-week history of painful swelling on the right side of her face. The pain worsens when she eats. Examination of the face shows a right-sided, firm swelling that is tender to palpation. Oral examination shows no abnormalities. Ultrasonography shows a stone located in a duct that runs anterior to the masseter muscle and passes through the buccinator muscle. Sialoendoscopy is performed to remove the stone. At which of the following sites is the endoscope most likely to be inserted during the procedure? Ultrasonography shows that the stone is located in the parotid duct.

Lateral to the second upper molar tooth The parotid duct emerges from the anterior edge of the parotid gland, turns medially at the anterior edge of the masseter muscle, and, after piercing through the buccinator muscle, enters the oral cavity lateral to the second upper molar. Sialolithiasis, which affects the parotid gland or duct in approx. 20% of cases, typically manifests with acute pain while eating and tender swelling of the affected gland, both of which are seen here. Risk factors include dehydration, certain medications (e.g., anticholinergics), and trauma. Sialolithiasis is usually treated conservatively, e.g., with NSAIDs and stimulation of salivary flow by gland massage or warm compresses. In severe cases or if conservative treatment fails, salivary stones can be removed via sialoendoscopy or open surgical procedures.

A 45-year-old man is brought to the emergency department because of severe abdominal pain for the past 2 hours. He has a 2-year history of burning epigastric pain that gets worse with meals. His pulse is 120/min, respirations are 22/min, and blood pressure is 60/40 mm Hg. Despite appropriate lifesaving measures, he dies. At autopsy, examination shows erosion of the right gastric artery. Perforation of an ulcer in which of the following locations most likely caused this patient's findings? This patient has a history of epigastric pain, suggesting peptic ulcer disease. Pain that gets worse with meals and erosion of the right gastric artery suggest that the ulcer is located in the stomach.

Lesser curvature of the stomach The lesser curvature of the stomach is supplied by the right and left gastric artery. The right gastric artery arises from the common hepatic artery, while the left gastric artery arises from the celiac trunk. Since this patient's ulcer has eroded the right gastric artery, it likely originated from the lower lesser curvature, which is a common location for peptic ulcers.

A 25-year-old woman comes to the physician for routine follow-up evaluation. She has a history of poorly-controlled asthma, for which she uses albuterol and fluticasone inhalers. Her mother died at 66 years of complications from emphysema. Laboratory analysis shows decreased levels of a protease inhibitor that inhibits elastin degradation. Which abdominal organ is most likely to be involved in the pathogenesis of this patient's condition? This woman's family history and decreased levels of α1-antitrypsin (protease inhibitor that inhibits elastin degradation) indicate a diagnosis of α1-antitrypsin deficiency (AATD).

Liver the liver, is visible intraperitoneally as an often triangular, homogenous grey, soft tissue density on the patient's right side on an axial CT scan. The liver plays a central role in the pathogenesis of AATD. Mutations in the AAT gene lead to the production of a misfolded protein within hepatocytes. The defective AAT cannot be secreted and aggregates, which causes hepatic damage and possibly cirrhosis and/or primary liver cancer. Histopathology shows the protein aggregations as PAS-positive, spherical inclusion bodies in periportal hepatocytes.

A 15-year-old girl is brought to the physician by her mother for a 2-day history of abdominal pain, nausea, vomiting, diarrhea, and decreased appetite. Her last menstrual period was 3 weeks ago. Her temperature is 37.6°C (99.7°F). Abdominal examination shows tenderness to palpation with guarding in the right lower quadrant. Laboratory studies show a leukocyte count of 12,600/mm3. Which of the following is the most likely underlying cause of this patient's condition? The incidence of this patient's condition peaks between 10-19 years of age.

Lymphatic tissue hyperplasia This patient most likely presents with acute appendicitis. Lymphatic tissue hyperplasia is the primary etiology of appendicitis in children; fecalith obstruction is the most common etiology in adults. Less common etiologies of appendicitis include obstruction due to foreign bodies, worm infestations, and tumors (e.g., carcinoid tumor). Obstruction within the appendix results in mucus and fluids stasis, allowing bacterial overgrowth and ultimately causing inflammation of the organ. Patients with appendicitis typically present with abdominal pain (dull migratory periumbilical pain that progresses to sharp right lower quadrant pain), fever, loss of appetite, nausea, and leukocytosis, as seen in this patient.

A 68-year-old man comes to the physician because of a 4-month history of bad breath and progressive difficulty swallowing solid food. Physical examination shows no abnormalities. An upper endoscopy is performed and a photomicrograph of a biopsy specimen obtained from the mid-esophagus is shown. Which of the following is the most likely explanation for the biopsy findings? The concentric eosinophilic lesions seen in this image are known as keratin pearls.

Neoplastic proliferation of squamous epithelium Neoplastic proliferation of squamous epithelium is the hallmark of squamous cell carcinoma (SCC) of the esophagus, which can manifest with progressive dysphagia and halitosis due to food particles retained in the esophagus. The histopathology involves clusters of keratin pearls destroying normal tissue structure in the upper two-thirds of the esophagus. It typically affects patients with risk factors such as long-term alcohol use, tobacco exposure, achalasia, and diets low in fruits and vegetables.

A 6-year-old boy is brought to the physician by his mother because of a 6-month history of mild episodic abdominal pain. The episodes occur every 1-2 months and last for a few hours. The pain is located in the epigastrium, radiates to his back, and is occasionally associated with mild nausea. His mother is concerned that his condition might be hereditary because his older sister was diagnosed with congenital heart disease. He is otherwise healthy and has met all developmental milestones. He is at the 75th percentile for height and the 65th percentile for weight. Physical examination shows no abdominal distention, guarding, or rebound tenderness. Which of the following congenital conditions would best explain this patient's symptoms? This patient's episodic nausea with epigastric pain that radiates to the back suggests recurrent acute pancreatitis.

Pancreas divisum Pancreas divisum is a congenital malformation that is caused by a failure of fusion of the ventral and dorsal pancreatic ducts, which normally join at 8 weeks of gestation. Pancreas divisum is present in about 10% of the population and is usually asymptomatic, but a minority of cases can develop chronic abdominal pain or recurrent episodes of acute pancreatitis, as seen here. Pancreas divisum can be diagnosed by endoscopic ultrasonography or MRCP, which would show a ventral pancreatic duct that drains along with the common bile duct into the major duodenal papilla and a dorsal pancreatic duct that drains separately into the duodenum via the minor duodenal papilla.

A 47-year-old woman with chronic epigastric pain comes to the physician because of a 1-month history of intermittent, loose, foul-smelling stools. She has also had a 6-kg (13-lb) weight loss. She has consumed 9-10 alcoholic beverages daily for the past 25 years. Seven years ago, she traveled to Mexico on vacation; she has not been outside the large metropolitan area in which she resides since then. She appears malnourished. The stool is pale and loose; fecal fat content is elevated. An immunoglobulin A serum anti-tissue transglutaminase antibody assay is negative. Further evaluation is most likely to show which of the following? The patient's chronic and heavy alcohol consumption is the key driver of a process that leads to fat malabsorption, which manifests as steatorrhea (bulky, greasy, foul-smelling stool due to increased fat content).

Pancreatic calcifications Chronic pancreatitis is the most likely diagnosis in a patient with a history of chronic epigastric pain and features of pancreatic insufficiency (e.g., steatorrhea, weight loss). Alcohol use disorder is the most common etiology of chronic pancreatitis, accounting for 60-70% of cases. Diagnosis is confirmed on abdominal CT imaging, which can show pancreatic calcifications, ductal strictures, and ductal dilatation. A "chain of lakes" appearance can sometimes be seen, which is caused by the alternating dilation and stricture of the main pancreatic duct. Pancreatic function tests (e.g., fecal elastase-1 measurement, 72-hour quantitative fecal fat test) can be used to assess the degree of pancreatic exocrine and enzyme deficiency.

A 58-year-old man comes to the physician because of a 4-day history of abdominal pain and vomiting. Initially, the vomitus was food that he had recently eaten, but it is now bilious. He has had similar complaints several times in the past 6 years. He has smoked 1 pack of cigarettes daily for the past 25 years and drinks 18 oz of whiskey daily. He is 160 cm (5 ft 3 in) tall and weighs 48 kg (105 lb); BMI is 19 kg/m2. His vital signs are within normal limits. Physical examination shows an epigastric mass. The remainder of the examination shows no abnormalities. Which of the following is the most likely diagnosis? This patient presents with recurrent abdominal pain, vomiting, and a palpable epigastric mass, while his vital signs and physical examination are otherwise unremarkable. His history of alcohol use suggests an underlying condition that is often caused by chronic alcohol use.

Pancreatic pseudocyst A pancreatic pseudocyst, which is a complication of acute or chronic pancreatitis, is caused by leakage of pancreatic exocrine secretions from damaged ducts. This patient's history of recurrent epigastric pain, vomiting, and chronic alcohol consumption makes a diagnosis of pancreatitis highly likely. Patients with chronic pancreatitis and heavy alcohol use are often cachectic due to malnutrition. Pancreatic pseudocysts, when large enough, can manifest with a palpable epigastric mass and bilious vomiting (due to extrinsic compression of the distal duodenum) or non-bilious vomiting (due to gastric outlet obstruction). The diagnosis can be confirmed by imaging; a well-defined fluid collection would be seen in the vicinity of the pancreas.

A 45-year-old woman with hypothyroidism comes to the physician because of progressive fatigue, lethargy, and epigastric pain after eating. Physical examination shows pale conjunctivae. Laboratory studies show decreased serum hemoglobin levels and increased serum gastrin levels. Esophagogastroduodenoscopy shows inflammation of the gastric body and fundus. A photomicrograph of a biopsy specimen taken from the gastric antrum is shown. Which of the following is the most likely cause of this patient's symptoms? This patient's biopsy sample shows antral G cell hyperplasia, which is consistent with a diagnosis of atrophic gastritis.

Parietal cell destruction Parietal cell destruction in the gastric fundus is seen in autoimmune-mediated atrophic gastritis (AMAG), which results in reduced gastric acid production. The reduced acid production in AMAG leads to loss of negative feedback on G cells, which subsequently causes G cell hyperplasia in the gastric antrum and hypergastrinemia. Parietal cell destruction also impairs intrinsic factor production and causes decreased vitamin B12 absorption in the ileum, leading to vitamin B12 deficiency anemia (pernicious anemia). However, iron deficiency anemia may be present earlier in the disease course, due to the decreased solubility of iron in the setting of hypochlorhydria.

A 43-year-old woman comes to the physician because of worsening heartburn and abdominal pain for the past 4 months. During this period she has also had multiple episodes of greasy diarrhea. Six months ago, she had similar symptoms and was diagnosed with a duodenal ulcer. Her mother died of complications from uncontrolled hypoglycemia and had primary hyperparathyroidism. The patient does not drink alcohol or smoke cigarettes. Her only medications are pantoprazole and ranitidine. Her epigastric region is tender when palpated. An esophagogastroduodenoscopy shows a friable ulcer in the distal duodenum. Further evaluation is most likely to show which of the following? Treatment-resistant peptic ulcer disease and features of malabsorption, combined with a family history suggestive of multiple endocrine neoplasia (primary hyperparathyroidism and pancreatic insulinoma), raises suspicion for Zollinger-Ellison syndrome (ZES).

Parietal cell hyperplasia in the stomach Parietal cell hyperplasia in the stomach can be seen in ZES, which involves a gastrin-secreting neuroendocrine tumor of the gastrointestinal tract. Hypergastrinemia leads to stimulation of parietal cells and excessive gastric acid production. As neuroendocrine tumors carry somatostatin receptors, radiolabelled octreotide can be used diagnostically to locate the tumor.

A 54-year-old woman comes to the emergency department because of a 5-hour history of diffuse, severe abdominal pain, nausea, and vomiting. She reports that there is no blood or bile in the vomitus. Two weeks ago, she started having mild aching epigastric pain, which improved with eating. Since then, she has gained 1.4 kg (3 lb). She has a 2-year history of osteoarthritis of both knees, for which she takes ibuprofen. She drinks 1-2 glasses of wine daily. She is lying supine with her knees drawn up and avoids any movement. Her temperature is 38.5°C (101.3°F), pulse is 112/min, respirations are 20/min, and blood pressure is 115/70 mm Hg. Physical examination shows abdominal tenderness and guarding; bowel sounds are decreased. An x-ray of the chest is shown. Which of the following is the most likely cause of this patient's current symptoms? The chest x-ray shows free air below the diaphragm.

Perforated peptic ulcer Perforation of a peptic ulcer can manifest with acute, diffuse abdominal pain, nausea, vomiting, fever, and tachycardia, all of which are seen in this patient. The chest x-ray finding of pneumoperitoneum further supports this diagnosis. Examination findings are typical for secondary peritonitis (abdominal guarding) and developing paralytic ileus (decreased bowel sounds). This patient's recent history of epigastric pain relieved by food intake in combination with regular alcohol consumption and ibuprofen use indicates an underlying duodenal ulcer. Perforation is the second most common complication of peptic ulcer disease (after gastrointestinal bleeding) and most often occurs in duodenal ulcers of the anterior wall, which are less protected than other types of ulcers by surrounding anatomical structures.

A 57-year-old woman comes to the physician because of a 2-week history of worsening epigastric pain that improves with meals. She has had similar pain of lesser intensity for the past 4 years. Physical examination shows no abnormalities. Upper endoscopy shows a 0.5-cm mucosal breach in the anterior duodenal bulb that extends into the submucosa. A biopsy specimen of the lesion shows hypertrophy of the Brunner glands. This patient is at the greatest risk for which of the following complications? This patient has epigastric pain that improves with food intake, which suggests a duodenal ulcer. The diagnosis of duodenal ulcer is confirmed by endoscopic findings, including a mucosal breach in the anterior duodenum, and by the hypertrophic Brunner glands in the biopsy specimen. The location of a peptic ulcer partly determines its most likely complications.

Perforation Peptic ulcer perforation occurs when an ulcer erodes through all the layers of the bowel, allowing spillage of gastric or duodenal contents into surrounding structures. It is the second most common complication of chronic PUD, bleeding being the first. The most common location for peptic ulcer perforation is the anterior duodenum. Ulcers of the anterior duodenum usually perforate into the peritoneal cavity, in contrast to ulcers of the posterior duodenum, which tend to cause massive bleeding (e.g., hematemesis) from the adjacent gastroduodenal artery.

A 28-year-old man comes to the physician because of a 6-month history of progressive fatigue and intermittent diarrhea. During this time, he has had a 6-kg (13-lb) weight loss. Physical examination shows pale conjunctivae. Abdominal examination shows tenderness to palpation in the lower quadrants. A photograph from a colonoscopy of the descending colon is shown. Further evaluation is most likely to show which of the following findings? This patient has chronic intermittent diarrhea, weight loss, fatigue, and lower abdominal pain, which are typical for an inflammatory bowel disease (IBD). The conjunctival pallor suggests anemia, possibly due to chronic gastrointestinal bleeding. The colonoscopy shows an erythematous and inflamed mucosa with fibrin-covered ulcerations, which are highly suggestive of ulcerative colitis.

Perinuclear antineutrophil cytoplasmic antibodies Perinuclear antineutrophil cytoplasmic antibodies (pANCA) are autoantibodies against myeloperoxidase granules in the perinuclear region of the cytoplasm. They are positive in up to 80% of patients with ulcerative colitis (presents as "lead pipe" on endoscopy), with no correlation between the titer and disease activity. pANCA can also be increased in patients with other inflammatory disorders, including autoimmune systemic vasculitis (especially Churg-Strauss syndrome and microscopic polyangiitis) and primary sclerosing cholangitis.

A 40-year-old woman comes to the physician because of a 2-week history of anal pain that occurs during defecation and lasts for several hours. She reports that she often strains during defecation and sees bright red blood on toilet paper after wiping. She typically has 3 bowel movements per week. Physical examination shows a longitudinal, perianal tear. This patient's symptoms are most likely caused by tissue injury in which of the following locations? The region of tissue injury in this patient receives innervation from the inferior rectal branch of the pudendal nerve.

Posterior midline of the anal canal, distal to the pectinate line Approximately 90% of anal fissures occur at the posterior midline of the anal canal, distal to the pectinate line (dentate line). The etiology is usually benign and primary in nature, with the most common cause being local trauma (e.g., passing of hard stools due to constipation), persistent diarrhea, vaginal delivery, or anal sex. This predilection for the posterior midline is likely due to poor perfusion to this area. Often, a skin tag (sentinel pile) formed by hypertrophied papillae would be visible in the perianal region at the site of the fissure.

A 21-year-old woman comes to the physician because of a 4-day history of abdominal cramps and bloody diarrhea 5 times per day. Her symptoms began after she ate an egg sandwich from a restaurant. Her vital signs are within normal limits. Physical examination shows diffuse abdominal tenderness. Stool culture shows gram-negative rods that produce hydrogen sulfide and do not ferment lactose. Which of the following effects is most likely to occur if she receives antibiotic therapy? Salmonella are gram-negative rods that produce hydrogen sulfide and do not ferment lactose. Nontyphoidal species of salmonella (e.g., Salmonella enterica) can cause dysentery. Uncooked poultry products are a common source of salmonella infections.

Prolonged fecal excretion of the pathogen Salmonella gastroenteritis may be treated with antibiotics, such as fluoroquinolones (e.g., ciprofloxacin). However, a potential side effect of this treatment is the prolonged fecal excretion of salmonella. The mechanism behind this is not known. Because of this side effect, antibiotics should only be used if clinically indicated and for an adequate duration. Antibiotics are only indicated for severe cases of salmonella gastroenteritis, which include diarrhea that occurs more than 9 times per day or the presence of systemic symptoms (e.g., fever, tachycardia, hypotonia). This patient only has local symptoms with normal vital signs, hence she would not qualify for antibiotics.

A 15-year-old boy is brought to the emergency department by his mother because of a 5-hour history of right lower quadrant pain, vomiting, and abdominal distention. Examination shows a palpable mass in the right lower quadrant of the abdomen. An x-ray of the abdomen shows a dilated ascending colon with an air-fluid level in the small intestine. A test is performed in which electrodes are placed on the nasal epithelium and the nose is perfused with several different solutions. When a chloride-free solution is administered, hyperpolarization across the nasal epithelium is absent. Which of the following is the most common cause of mortality in patients with the condition described here? This patient has clinical findings consistent with distal intestinal obstruction syndrome (abdominal distention, RLQ pain, and a RLQ mass), most likely due to abnormal chloride transmembrane transport.

Pulmonary infection The nasal potential difference test conducted here confirms the diagnosis of cystic fibrosis in this patient. Patients with cystic fibrosis have hyperviscous exocrine gland secretions, which can thicken stool and cause distal intestinal obstruction, as seen here. Hyperviscous respiratory secretions and deficient mucociliary clearance lead to recurrent pulmonary infections. This, combined with multiple episodes of antibiotic therapy, leads to the development of more severe, resistant infections (e.g., Pseudomonas aeruginosa) and structural lung changes (e.g., bronchiectasis, emphysema), with eventual progression to respiratory failure.

A 10-day-old male newborn is brought to the physician by his mother because of difficulty feeding and frequent nonbilious vomiting. His stool is soft and yellow-colored. The pregnancy was complicated by polyhydramnios and results from chorionic villus sampling showed a 47, XY, +21 karyotype. Physical examination shows mild abdominal distention and normal bowel sounds. An x-ray of the abdomen with oral contrast is shown. The most likely cause of his condition is due to a defect in which of the following embryologic processes? The karyotype of this patient has an additional chromosome 21 with a total of 47 chromosomes, indicating Down syndrome. Abdominal x-ray shows the double bubble sign with contrast in the stomach and the proximal duodenum.

Rotation of the ventral pancreatic bud During embryologic development of the pancreas, the endoderm forms two buds: a ventral and a dorsal one. In the process of foregut rotation, the ventral pancreatic bud normally fuses with the dorsal pancreatic bud. A defect in this process can lead to the fusion of abnormally rotated buds, resulting in a ring of pancreatic tissue surrounding the duodenum (annular pancreas) and duodenal stenosis. In the antenatal period, this manifests as polyhydramnios and in the neonatal period, it manifests as feeding intolerance, vomiting (typically non-bilious, but can be bilious depending on whether the obstruction is proximal or distal to the major duodenal papilla), and abdominal distension. Annular pancreas is associated with Down syndrome.

A 49-year-old man with HIV comes to the physician because of a 1-month history of intermittent diarrhea and abdominal pain. Abdominal examination shows mild, diffuse tenderness throughout the lower quadrants. His CD4+ T-lymphocyte count is 180/mm3. Colonoscopy shows multiple hemorrhagic nodules in the rectum and descending colon. Polymerase chain reaction of the lesions is positive for HHV-8. Histologic examination of the lesions is most likely to show which of the following findings? This HIV-positive patient with a very low CD4+ T-lymphocyte count (< 200/mm3) presents with HHV-8-positive lesions, making Kaposi sarcoma the most likely diagnosis.

Spindle-shaped cells with lymphocytic infiltration Vascular lesions showing spindle-shaped cells with lymphocytic infiltration are consistent with Kaposi sarcoma, an AIDS-defining condition that is caused by HHV-8 infection. Kaposi sarcomas originate from endothelial cells and usually manifest with asymptomatic vascular tumors of the skin. These tumors can also involve the lymph nodes and gastrointestinal tract, potentially causing diarrhea and abdominal pain. Additional histologic features of Kaposi sarcoma include slit-like vascular spaces and extravasated erythrocytes.

An otherwise healthy 45-year-old woman comes to the physician because of a 1-year history of episodic abdominal cramps, bloating, and flatulence. The symptoms worsen when she has pizza or ice cream and have become more frequent over the past 4 months. Lactose intolerance is suspected. Which of the following findings would most strongly support the diagnosis of lactose intolerance? Lactose intolerance does not cause bowel wall inflammation, so normal histological architecture is expected on biopsy.

Tall villi with focal collections of goblet cells Tall villi lined by simple columnar epithelium and crypts with interspaced goblet cells (crypts of Lieberkühn) and Paneth cells describes normal jejunal architecture. Patients with lactose intolerance are deficient in lactase, permitting osmotically active lactose to pass undigested to the large bowel. The molecule subsequently binds water and acts as a substrate for colonic bacteria, leading to symptoms of flatulence, bloating, and abdominal pain. Biopsy shows normal histological findings, but samples will also show decreased activity of lactase.

A 19-year-old woman comes to the physician because of a 1-year history of severe abdominal pain, bloating, and episodic diarrhea. She has also had a 10-kg (22-lb) weight loss over the past 10 months. Physical examination shows a mildly distended abdomen, diffuse abdominal tenderness, and multiple erythematous, tender nodules on the anterior aspect of both legs. There is a small draining lesion in the perianal region. Further evaluation of this patient's gastrointestinal tract is most likely to show which of the following findings? Recurrent episodes of severe abdominal pain, diarrhea, and a perianal fistula with drainage in combination with erythema nodosum (multiple erythematous, tender nodules on the legs) are suggestive of a diagnosis of Crohn disease.

Transmural inflammation Transmural intestinal inflammation is a characteristic finding of Crohn disease. It commonly results in abscess and fistula formation, as seen in this patient. Endoscopy confirms the diagnosis, showing a discontinuous pattern of involvement with linear ulcers ("snail trails") and a cobblestone appearance of the affected GI areas (typically terminal ileum and colon). Biopsy reveals inflammation of all intestinal layers and noncaseating granulomas.

A 34-year-old man comes to the physician because of palpitations, shortness of breath, diarrhea, and abdominal cramps for 2 months. Physical examination shows cutaneous flushing of the face. Auscultation of the chest shows bilateral wheezing. A 24-hour urine collection shows increased 5-hydroxyindoleacetic acid (5-HIAA) concentration. A contrast-enhanced CT scan of the abdomen shows an intestinal tumor with extensive metastasis to the liver. A diagnosis of an inoperable disease is made and the patient is started on treatment with octreotide. Six weeks later, the patient's symptoms have improved except for his abdominal pain and frequent loose stools. The physician suggests enrolling the patient in a trial to test additional treatment with a new drug that has been shown to improve symptoms in other patients with the same condition. The expected beneficial effect of this new drug is most likely caused by inhibition of which of the following? Palpitations, dyspnea with wheezing, flushing, and diarrhea in a patient with a metastatic intestinal tumor and increased urinary 5-HIAA is suggestive of carcinoid syndrome. Telotristat is used as adjunctive therapy in patients with carcinoid tumors.

Tryptophan hydroxylase Tryptophan hydroxylase is an enzyme that converts tryptophan to serotonin. The hallmark of carcinoid tumors is the excess production of serotonin, which is primarily responsible for the gastrointestinal symptoms (e.g., diarrhea) in carcinoid syndrome. Telotristat, an inhibitor of tryptophan hydroxylase, has recently been approved as adjunctive therapy for patients with carcinoid tumors whose diarrhea is not controlled with somatostatin analogs (e.g., octreotide).


Related study sets

CCM NUR 121- UNIT 1CCM Nursing Program

View Set

Test #3 - Chapter 44: Assessment of Digestive and Gastrointestinal Function

View Set

Maternal Newborn Success Chapter 7

View Set

Endocrine ch. 16 Bio 116 (Nicholls)

View Set

Liam Tvenstrup Academic Team Part 1

View Set

2000 SAT Words with tumblrs and pictures

View Set

Slope Intercept Form a.k.a. y = mx + b

View Set

The Age of Exploration Geography and History

View Set

Med Surg Respiratory Care Modalities

View Set